You are on page 1of 62

TEST – 24

(3808)

FULL LENGTH TEST


(COMPLETE SYLLABUS):

Indian Constitution, Political System and Governance +


Geography + History of India + Indian Economy & Social
Development + Ecology & Environment + General Science +
Current Affairs

2023
INDIAN COPIERS SYSTEMS

ALL KIND STUDY MATERIAL OF


IAS, PCS, SSC, IGNOU, JUDICIARY
ETC.
ALL TEST SERIES AVAILABLE

CONTACT US AT:
Telegram or Contact - 9953078429
or
Call at 9818130472

011- 44786293
EMAIL US:
Indiancopiers2018@gmail.com
Also Visit at : www.indiancopiers.com
COURIER FACILITY ALSO AVAILABLE ALL OVER INDIA
VISIONIAS
www.visionias.in

Test Booklet Series

TEST BOOKLET

GENERAL STUDIES (P) 2023 – Test – 3808


C
Time Allowed: Two Hours Maximum Marks: 200

INSTRUCTIONS

1. IMMEDIATELY AFTER THE COMMENCEMENT OF THE EXAMINATION, YOU SHOULD CHECK THAT THIS BOOKLET
DOES NOT HAVE ANY UNPRINTED OR TURN OR MISSING PAGES OR ITEMS, ETC. IF SO, GET IT REPLACED BY A
COMPLETE TEST BOOKLET.

2. ENCODE CLEARLY THE TEST BOOKLET SERIES A, B, C OR D AS THE CASE MAY BE IN THE APPROPRIATE PLACE IN
THE ANSWER SHEET.

3. You have to enter your Roll Number on the Test Booklet in the Box
provided alongside. Do NOT write anything else on the Test Booklet.

4. This Test Booklet contains 100 items (Questions). Each item is printed in English. Each item comprises four
responses (answers). You will select the response which you want to mark on the Answer Sheet. In case you feel
that there is more than one correct response with you consider the best. In any case, choose ONLY ONE response
for each item.

5. You have to mark all your responses ONLY on the separate Answer Sheet provided. See direction in the answers
sheet.

6. All items carry equal marks. Attempt all items. Your total marks will depend only on the number of correct
responses marked by you in the answer sheet. For every incorrect response 1/3rdof the allotted marks will be
deducted.

7. Before you proceed to mark in the Answer sheet the response to various items in the Test booklet, you have to
fill in some particulars in the answer sheets as per instruction sent to you with your Admission Certificate.

8. After you have completed filling in all responses on the answer sheet and the examination has concluded, you
should hand over to Invigilator only the answer sheet. You are permitted to take away with you the Test Booklet.

9. Sheet for rough work are appended in the Test Booklet at the end.

DO NOT OPEN THIS BOOKLET UNTIL YOU ARE ASKED TO DO SO

1 www.visionias.in ©Vision IAS


1. The legislature of a state can impose 4. Tooth enamel is the hardest substance in the
reasonable restrictions on the freedom of body. It does not dissolve in water but is
trade, commerce, and intercourse with that
corroded when the pH in the mouth is below
state or within that state in the public interest.
A bill for this purpose can be introduced in 5.5. It is made up of :
the legislature only with the previous sanction (a) Calcium phosphate
of the
(b) Calcium Oxide
(a) Governor of the state
(b) Speaker of the legislative Assembly (c) Potassium Hydroxide
(c) The President of India (d) Potassium Chloride
(d) Parliament

2. Consider the following statements with 5. The term ‘synNotch’ is sometimes seen in the
reference to elections in the Panchayati Raj news with reference to
system:
(a) CAR T-cell Therapy
1. The minimum age for contesting in the
elections to the Panchayats is 25 years. (b) Blockchain technology
2. As per the Constitution (Seventy-third (c) Artificial neural networks
Amendment) Act, 1992, any questions of
(d) E-commerce
disqualifications shall be strictly referred
to the high court of the state.
Which of the statements given above is/are 6. Consider the following statements regarding
correct?
the profile of the agriculture sector in India:
(a) 1 only
1. The share of the agriculture & allied
(b) 2 only
(c) Both 1 and 2 sectors in the total GVA of the economy
(d) Neither 1 nor 2 has consistently been higher than 20 per

cent.
3. Consider the following statements with
reference to death penalty and mercy petition: 2. Private investment accounts for majority
1. In 1980, in the Bachan Singh vs. State of of share in Gross Capital Formation
Punjab case, the Supreme Court of India
(GCF) in agriculture and allied sectors.
upheld the constitutional validity of the
death penalty. 3. More than 80% of rural households in
2. Article 72 of the Indian Constitution India are engaged in agriculture.
clearly defines the procedure related to
Which of the statements given above is/are
mercy petitions against death penalties.
Which of the statements given above is/are correct?
not correct? (a) 1 and 2 only
(a) 1 only
(b) 2 only
(b) 2 only
(c) Both 1 and 2 (c) 1 and 3 only

(d) Neither 1 nor 2 (d) 1, 2 and 3


2 www.visionias.in ©Vision IAS
7. Consider the following statements regarding 10. Consider the following pairs:
the Mhadei (Mandovi) river: Schemes Objectives
1. It originates in the Western Ghats in
1. Net : To receive energy credit
Karnataka.
metering by feeding excess solar
2. It is a tributary of the Malaprabha River.
policy power generated from
3. The famous Dudhsagar waterfall is
located on the river. solar panels to grid
Which of the statements given above is/are 2. KUSUM : Encourage farmers to
correct? generate income by
(a) 2 only
selling surplus power to
(b) 1 and 2 only
the grid
(c) 1 and 3 only
3. SRISTI : Incentivise installation of
(d) 1, 2 and 3
Scheme rooftop solar power plant
8. With reference to the 'Dharamshala 4. Atal Jyoti : Illuminate dark regions
declaration', sometimes seen in the news, through establishment of
Yojana
consider the following statements:
solar street lights
1. It aims to recover tourism in India to the
How many pairs given above are correctly
pre-pandemic level by 2024.
2. It includes a long-term revenue goal of $1 matched?

trillion by the tourism sector of India by (a) Only one pair


2047. (b) Only two pairs
Which of the statements given above is/are (c) Only three pairs
correct?
(d) All four pairs
(a) 1 only
(b) 2 only
(c) Both 1 and 2 11. In the context of marine biodiversity, which

(d) Neither 1 nor 2 of the following are phytoplankton?


1. Diatoms
9. Consider the following: 2. Dinoflagellates
1. Variable Rate Applicator
3. Crustaceans
2. Geographic Information Systems (GIS)
4. Coccolithophores
3. Remote Sensing (RS) technique
4. Global Navigation Satellite Systems 5. Foraminifera

(GNSS) Select the correct answer using the code given


Which of the above technologies are used in below.
precision farming? (a) 1, 2 and 4 only
(a) 1 and 2 only
(b) 1, 3 and 5 only
(b) 2, 3 and 4 only
(c) 2, 4 and 5 only
(c) 1, 3 and 4 only
(d) 1, 2, 3 and 4 (d) 1, 2, 3, 4 and 5
3 www.visionias.in ©Vision IAS
12. The Second Schedule of the Indian 15. As per extant FDI Policy, 100% FDI is
Constitution contains the provisions relating allowed in which of the following sectors of
to the emoluments, allowances, and agriculture through automatic route?
privileges for different offices. In this 1. Development and Production of seeds
context, which of the following offices is not 2. Services related to agro and allied sectors

covered under the Second Schedule? 3. Palm Oil plantations

(a) Judges of the high courts 4. Multi-Brand retail in the agricultural

(b) Speakers of legislative assemblies sector


Select the correct answer using the code given
(c) Governor of states
below.
(d) Chief ministers of states
(a) 1, 2 and 3 only
(b) 3 and 4 only
13. Which of the following statements is correct
(c) 1, 2 and 4 only
regarding the Investor Education and
(d) 1, 2, 3 and 4
Protection Fund Authority (IEPFA)?
(a) It is an autonomous body set up under the
16. With reference to the religious history of
aegis of the Ministry of Finance.
India, Mahavibhasa is a
(b) It has been established under the
(a) book dealing with rules of sacrifice used
provisions of the Securities and
by Brahmins.
Exchange Board of India Act, 1992.
(b) rules of conduct for Digambar jains.
(c) It is a quasi-judicial body set up under the (c) summary of doctrines of Sarvastivadin
Insolvency and Bankruptcy Code, 2016. sect of Buddhists.
(d) It has been set up to protect the interests (d) book on religious plays written by Bhasa.
of investors under the Companies Act,
2013. 17. Consider the following statements regarding
coal industry in India:
14. Consider the following statements with 1. India is the world's second-largest
reference to parliamentary committees: producer and consumer of coal.
1. The Constitution of India has prescribed 2. Coal India Limited (CIL) contributes
specific functions for all parliamentary more than 80% of the indigenous
committees. production/supply of coal.

2. The standing committees are permanent 3. Coal production has a weightage of 40%

and work on a continuous basis. in the Index of Core Industries.

Which of the statements given above is/are Which of the statements given above is/are

not correct? correct?


(a) 1 and 2 only
(a) 1 only
(b) 2 and 3 only
(b) 2 only
(c) 1 only
(c) Both 1 and 2
(d) 1, 2 and 3
(d) Neither 1 nor 2
4 www.visionias.in ©Vision IAS
18. Consider the following statements regarding 21. Consider the following statements with
Gaur or Indian Bison:
reference to Law Commission:
1. It is endemic to India.
2. It is categorised as critically endangered 1. Law Commission is a statutory body
species under the IUCN Red List of constituted by the Government of India.
Threatened Species.
3. It can be found in Bandipur National 2. The first law commission in Independent
Park. India was established in 1955 with M. C.
Which of the statements given above is/are
Setalvad as its chairman.
correct?
(a) 1 and 3 only 3. No Law Commission has been
(b) 2 only
constituted since the year 2018.
(c) 2 and 3 only
(d) 3 only Which of the statements given above is/are

correct?
19. India’s public sector capital expenditure is
likely to fall to an eight-year low. In this (a) 1 and 2 only
context, consider the following statements: (b) 1, 2 and 3
1. Capital expenditure as a percentage of
GDP by the centre declined sharply (c) 2 only
during the last year. (d) 3 only
2. Capital spending by central public sector
enterprises (CPSEs) increased for the
third consecutive year. 22. Which of the following statements are correct
Which of the statements given above is/are
with respect to the ‘Prati Sarkar’ established
correct?
(a) 1 only in Satara during the Quit India Movement?
(b) 2 only
1. It was the first parallel government set up
(c) Both 1 and 2
(d) Neither 1 nor 2 during the Quit India Movement.

2. It established an armed contingent called


20. Consider the following statements regarding
pyrolysis: Vidyut Vahini.
1. It is the thermal degradation of plastic and 3. Nyayadan Mandals were set up for the
biomass waste at different temperatures,
dispensation of justice.
in the presence of oxygen.
2. Pyrolysis of carbonaceous wastes like 4. Gandhi marriages were conducted to
coconut shell yields charcoal.
abolish untouchability.
3. It is considered as a cleaner technology in
waste management compared to Select the correct answer using the code given
incineration. below.
Which of the statements given above is/are
correct? (a) 1, 2 and 3 only
(a) 2 only (b) 1, 3 and 4 only
(b) 1 and 3 only
(c) 3 and 4 only
(c) 1 and 2 only
(d) 2 and 3 only (d) 1, 2 and 4 only
5 www.visionias.in ©Vision IAS
23. Consider the following statements with 25. Consider the following statements regarding

respect to Inflationary Gap: Quality Control Orders (QCOs):

1. It is the gap between GDP growth rates of 1. QCOs are issued by various Ministries

two consecutive years. under the Bureau of Indian Standards

Act, 2016.
2. In this condition, the potential GDP must
2. The items, under these orders, cannot be
be higher than the current real GDP.
produced, traded and stocked unless they
3. Reduction in government spending could
bear the BIS mark.
help reducing this gap.
3. These orders also help in containing
Which of the statements given above is/are
imports of non-essential items.
correct?
Which of the statements given above is/are
(a) 1 and 2 only
correct?
(b) 3 only
(a) 1 only
(c) 2 and 3 only (b) 2 and 3 only
(d) 1, 2 and 3 (c) 1 and 3 only

(d) 1, 2 and 3

24. Consider the following statements regarding

stem cells: 26. The Crypto-Asset Reporting Framework

1. Multipotent stem cells are able to divide (CARF) for reporting and exchange of

and differentiate into cells of the whole information with respect to crypto-assets was

released by
organism.
(a) World Bank Group
2. Pluripotent stem cells (PSC) form cells of
(b) World Economic Forum
all germ layers but not extraembryonic
(c) World Trade Organization
structures, such as the placenta.
(d) Organisation for Economic Co-operation
3. Totipotent stem cells have a narrower
and Development
spectrum of differentiation than PSCs.

Which of the statements given above is/are


27. The term “Kessler Syndrome” is talked about
correct?
in the context of:
(a) 1 and 2 only (a) Climate change
(b) 2 only (b) Space debris
(c) 1 and 3 only (c) Genetically Modified (GM) crops

(d) 1, 2 and 3 (d) Drone warfare


6 www.visionias.in ©Vision IAS
28. With reference to corals, consider the 30. Consider the following pairs:

following statements: Term in the news Entity


1. WALLABY : Radio telescope
1. Corals are marine invertebrates
2. Gaia BH1 : Satellite
belonging to the phylum Cnidaria. 3. RAD12 : Galaxy
2. Black corals do not have symbiotic algae Which of the pairs given above is/are

associated with them and typically found correctly matched?


(a) 1 only
at greater depths of oceans.
(b) 1 and 3 only
3. Many species of corals have growth rings (c) 2 and 3 only
that can be used to reconstruct past (d) 1, 2 and 3

climate.
31. Consider the following statements regarding
Which of the statements given above is/are
the Arctic Council:
correct? 1. It was set up as per the Ottawa
(a) 1 and 3 only Declaration of 1996.

(b) 2 only 2. It works for promoting cooperation,


coordination, and interaction among the
(c) 2 and 3 only
Arctic States and Arctic Indigenous
(d) 1, 2 and 3 people.
3. India is a member of the Arctic Council.

29. With regard to the Provisional Government of Which of the statements given above is/are
correct?
Free India set up in 1943, consider the
(a) 1 and 2 only
following statements: (b) 2 only
1. It was set up by Captain Mohan Singh in (c) 1 only

Singapore. (d) 1, 2 and 3

2. Subhash Chandra Bose was declared the


32. Consider the following statements regarding
Prime Minister of this provisional oil palm:
government. 1. It can tolerate two to four months of dry

3. The provisional government was spells.


2. It has a short gestation period that starts
recognized by axis powers and it declared
yielding from the first year of its planting.
war on allied powers. 3. India is the second biggest importer of
Which of the statements given above is/are palm oil after China.

correct? Which of the statements given above is/are


correct?
(a) 1 and 3 only
(a) 1 only
(b) 2 only (b) 1 and 2 only
(c) 2 and 3 only (c) 1, 2 and 3

(d) None (d) 2 and 3 only


7 www.visionias.in ©Vision IAS
33. Consider the following statements with 35. Consider the following pairs:
regard to the Company’s commercial Places in the news Country
privileges: 1. Sharm el-sheikh : Qatar
2. Samarkand : Uzbekistan
1. The ‘Golden Farman’ issued to the
3. Kherson : Iran
English Company by the Sultan of
4. Panjsheer : Afghanistan
Golconda in 1732, earned them the
Which of the pairs given above is/are
privilege of trading freely in the ports of
correctly matched?
Golconda. (a) 1 and 2 only
2. The Charter Act of 1793, paved the way (b) 4 only
for shipments of opium to China. (c) 2 and 4 only
3. The Company’s monopoly over trade (d) 2, 3 and 4 only

with China and in tea ended with the


36. In the context of the cultural history of India,
Charter Act of 1853.
consider the following statements regarding
Which of the statements given above is/are
Nettur Petti:
correct?
1. It is a handcrafted, ornamental box
(a) 1 only created by traditional craftsmen of
(b) 1 and 2 only Kerala.
(c) 2 only 2. It is completely made of sandalwood.
(d) 1, 2 and 3 Which of the statements given above is/are
correct?

34. Consider the following statements regarding (a) 1 only


(b) 2 only
the Global Tax Agreement framed by the
(c) Both 1 and 2
OECD:
(d) Neither 1 nor 2
1. The agreement tries to give more taxing
rights on multinational companies to the 37. Consider the following statements regarding
governments of their home countries. the appointment of a person as the governor
2. Under this agreement, countries agreed to of a state:
implement a minimum tax rate of 15% on 1. Any Indian citizen to be appointed as the
large multinational companies. governor of a state should have attained

3. Implementation of minimum tax rate the age of 35 years.


2. As a convention, the President is required
would lead to decline in global tax
to consult with the chief minister of state
revenues.
while appointing the Governor.
Which of the statements given above is/are
Which of the statement given above is/are
correct? correct?
(a) 1 and 2 only (a) 1 only
(b) 2 only (b) 2 only
(c) 1 and 3 only (c) Both 1 and 2
(d) 1, 2 and 3 (d) Neither 1 and 2
8 www.visionias.in ©Vision IAS
38. In the context of Kisan movements during the 41. Which of the following are among the four
freedom struggle, the South Indian stated pillars of the Indo-Pacific Economic
Federation of Peasants and Agricultural Framework?
1. Supply Chain
Labour was organized by
2. Clean Energy
(a) N.G. Ranga and E.M.S. Namboodripad
3. Containment of China
(b) Yadunandan Sharma and N.G. Ranga
4. Promotion of Democracy
(c) A. K. Gopalan and E. K. Nayanar 5. Anti-Corruption
(d) P. Krishna Pillai and Sahajanand 6. Trade
Saraswati 7. Military cooperation
Select the correct answer using the code given
39. Consider the following statements: below.
1. Only the Central Government is (a) 1, 2, 3 and 7 only
(b) 1, 2, 3, 5 and 7 only
empowered to notify an area as a
(c) 3, 4 and 6 only
Biodiversity Heritage Site.
(d) 1, 2, 5 and 6 only
2. Bambarde Myristica Swamps is a
Biodiversity Heritage Site located in the 42. Consider the following pairs:
state of Tamil Nadu. Jain Tirthankara Associated symbols
Which of the statements given above is/are 1. Rishabhnatha : Lotus
correct? 2. Parshavanatha : Serpent
(a) 1 only 3. Mahavira : Lion
(b) 2 only Which of the pairs given above is/are
(c) Both 1 and 2 correctly matched?
(d) Neither 1 nor 2 (a) 1 and 2 only
(b) 2 and 3 only
(c) 3 only
40. Which of the following statements are correct
(d) 1, 2 and 3
regarding India’s external debt?
1. US dollar-denominated debt is the largest
43. Consider the following statements about the
component with a share of more than 50 Chairman of the Rajya Sabha:
per cent. 1. He cannot preside over a sitting of the
2. Loans are the largest component with a Rajya Sabha when a resolution for his
share of more than 30 per cent. removal is under consideration.
3. Long term debt, with original maturity of 2. He cannot cast a vote even in the case of
above one year, constitutes more than 90 an equality of votes.
3. He does not preside over a joint sitting of
per cent share.
the two Houses of Parliament.
Select the correct answer using the code given
Which of the statements given above is/are
below.
correct?
(a) 1 and 3 only (a) 1 and 3 only
(b) 1 and 2 only (b) 3 only
(c) 2 and 3 only (c) 1 and 2 only
(d) 1, 2 and 3 (d) 1, 2 and 3
9 www.visionias.in ©Vision IAS
44. With reference to the representation in Rajya 47. In the context of economics, ‘Soft landing’ is:
Sabha, consider the following statements: (a) a cyclical slowdown in the rate of
deflation that avoids hyperinflation.
1. Currently, the Rajya Sabha has 250
(b) a stage between the expansion and the
members.
peak stage of an economic cycle.
2. All union territories of India are (c) a marked economic slowdown or
represented in the Rajya Sabha. downturn following a period of rapid
3. Among the union territories, Jammu and growth.

Kashmir has the highest number of Rajya (d) a cyclical slowdown in economic growth
that avoids recession.
Sabha seats.
Which of the statements given above is/are
48. Consider the following statements with
correct? reference to the aerosols:
(a) 3 only 1. Compared to Green House Gases
(b) 1 and 2 only (GHGs), aerosols are short-lived and
persist in the atmosphere only for a few
(c) 1 and 3 only
weeks.
(d) 1, 2 and 3
2. Aerosols serve as cloud condensation
nuclei resulting in high incidence of
45. The Coriolis force impacts which of the rainfall.
following things on earth? 3. Aerosols in the lower atmosphere can

1. Military Snipers modify the size of cloud particles


affecting Earth's energy budget.
2. Flight navigation
Which of the statements given above are
3. Trade winds
correct?
4. Cyclones (a) 1 and 2 only
Select the correct answer using the code given (b) 2 and 3 only
below. (c) 1 and 3 only

(a) 1, 2 and 3 only (d) 1, 2 and 3

(b) 3 and 4 only


49. Consider the following pairs:
(c) 2 and 4 only
Tribe Characteristic
(d) 1, 2, 3 and 4 1. Hausas : Settled cultivators
2. Eskimos : Live in compact igloos
46. In the context of biodiversity, Great Knot, 3. Zulus : Speak Bantu language
How many pairs given above are correctly
Indian Skimmer, Wilson’s Storm Petrel are
matched?
(a) birds
(a) None of the pairs
(b) fish (b) Only one pair
(c) turtles (c) Only two pairs
(d) butterflies (d) All three pairs
10 www.visionias.in ©Vision IAS
50. Consider the following statements with 53. In the context of evolution of science and
respect to Rashtriya Gram Swaraj Abhiyan: technology during medieval India, Yantraja
1. It aims to strengthen Panchayati Raj is a/an
Institutions with an emphasis on (a) astronomical instrument
Aspirational Districts. (b) hydraulic wheel to draw water

2. It is a central sector scheme. (c) type of lock

3. The scheme is implemented by the (d) coin-making device

Ministry of Rural Development.


Which of the statements given above is/are 54. Consider the following statements regarding
the black soils:
correct?
1. They are poor in fertility due to their
(a) 1 and 2 only
calcareous content.
(b) 2 and 3 only
2. They constitute less than 10% of global
(c) 1, 2 and 3
soils.
(d) 1 only
3. Their black colour is a result of the
accumulation of organic matter through
51. The Digital Market Act and the Digital
melanization process.
Services Act, sometimes seen in news, are
Which of the statements given above is/are
related to which of the following?
correct?
(a) European Union laws for safer digital
(a) 1 only
space
(b) 1 and 3 only
(b) G20 agreement on global digital
(c) 2 and 3 only
companies
(d) 3 only
(c) OECD laws to tackle cyber crimes
(d) 5G related laws by ASEAN 55. Consider the following statements regarding
Influenza viruses:
52. ‘Nai Chetna-Pahal Badlav Ki’ campaign is a: 1. They have a double-stranded segmented
(a) new scheme to increase women's DNA genome.
employment in the armed forces by the 2. It is transmitted primarily by droplets or
Ministry of Defence. respiratory secretions of infected persons.
(b) new scheme to provide collateral-free 3. Influenza D viruses primarily affect cattle
finance to self-help groups by the and are not known to infect or cause
Ministry of Finance. illness in people.
(c) new community-led national campaign Which of the statements given above is/are
by the Ministry of Rural Development, correct?
against gender-based discrimination. (a) 1 only

(d) new educational initiative by the Ministry (b) 2 only

of Education to increase literacy rates in (c) 2 and 3 only

older women. (d) 1, 2 and 3

11 www.visionias.in ©Vision IAS


56. Consider the following statements: 59. Consider the following statements regarding
1. Tax elasticity refers to the responsiveness the Intergovernmental Science-Policy
of tax revenue growth to changes in the Platform on Biodiversity and Ecosystem
GDP of an economy. Services (IPBES):
1. The United Nations Environment
2. Tax buoyancy refers to changes in tax
Programme (UNEP) provides secretariat
revenue in response to changes in tax
services to IPBES.
rates.
2. The Economics of Ecosystems and
Which of the statements given above is/are Biodiversity (TEEB) was conceptualized
correct? by IPBES.
(a) 1 only 3. India is a member of IPBES.
(b) 2 only Which of the statements given above is/are
(c) Both 1 and 2 correct?
(d) Neither 1 nor 2 (a) 1 only
(b) 1 and 3 only
(c) 2 and 3 only
57. Which of the following were advocated by
(d) 1, 2 and 3
the Nehru Report of 1928?
1. Universal adult suffrage
60. With reference to 5G communications,
2. Equal rights for women
consider the following statements:
3. Freedom to form unions 1. It can operate in all three band spectrums.
Select the correct answer using the code given 2. It places special emphasis on low latency
below. and energy efficiency.
(a) 1 and 2 only 3. 5G cell tower coverage is less extensive
(b) 2 and 3 only than 4G.
(c) 1 and 3 only Which of the statements given above are
(d) 1, 2 and 3 correct?
(a) 1 and 2 only
(b) 2 and 3 only
58. Consider the following statements with
(c) 1 and 3 only
regard to the Finance Commission mentioned
(d) 1, 2 and 3
under Article 280 of the Indian Constitution:
1. It is mandatory for the Parliament to set 61. Which of the following statements is correct
up a finance commission at the expiry of regarding Forked Fanwort, a plant species,
every fifth year. often mentioned in the news?
2. It should consist of a chairman and not (a) It is a plant species endemic to the
more than three other members. Western Ghats and Andaman & Nicobar
3. Qualifications of the members of the islands.
(b) It is an aquatic plant which has resurfaced
Commission are determined by the
in India after more than 1000 years.
Parliament.
(c) It is one of the rare species of frog found
Which of the statements given above is/are
on the Nicobar island, which is currently
correct? threatened by habitat loss.
(a) 1 and 2 only (d) It is a submerged perennial aquatic plant
(b) 1 and 3 only that grows in stagnant to slow flowing
(c) 2 and 3 only fresh water and invades the area where it
(d) 3 only grows.
12 www.visionias.in ©Vision IAS
62. He was sent by the Persian ruler as an 65. Consider the following statements with
emissary to Vijayanagar. He wrote his travels
regard to the Suhrawardi Silsilah:
in the Matla-us-Sadain wa Majma-ul-
Bahrain, or The Rise of Two Auspicious 1. In India, it was established by Sheikh
Constellations and the Confluence of Two
Bahauddin Zakariya.
Oceans.
Which of the following travellers has been 2. Suhrawardis maintained close contact
described by the passage given above?
with the state.
(a) Ibn Battuta
(b) Abdur Razzak Which of the statements given above is/are
(c) Ferishta
correct?
(d) Al-Biruni
(a) 1 only
63. This Gupta ruler conquered places and
(b) 2 only
unified a greater part of India under him. His
prestige and influence were so huge that the (c) Both 1 and 2
ruler of Sri Lanka Meghavarman, sent a
(d) Neither 1 nor 2
missionary seeking his permission to build a
Buddhist monastery at Gaya. His military
greats are mentioned in the accounts of his
66. With reference to plant and animal
court poet Harisena.
Which of the ruler is being described in the biodiversity, consider the following
passage given above?
statements:
(a) Samudragupta
(b) Chandragupta II 1. More than 50 percent of all the species
(c) Skandagupta
recorded are plants.
(d) Chandragupta 1
2. Among animals, insects make up more
64. Arrange the following electoral reforms in
than 70 percent of the total species.
India in chronological order.
1. Supply of electoral rolls free of cost 3. The number of fungi species in the world
2. Provision made to facilitate the use of
is more than the combined total of the
Electronic Voting Machines (EVMs) in
elections species of fishes and mammals.
3. Restrictions imposed on exit polls
Which of the statements given above is/are
4. Provision made to facilitate voting
through postal ballot correct?
Select the correct answer using the code given
(a) 1 only
below.
(a) 1-2-3-4 (b) 2 and 3 only
(b) 2-1-4-3
(c) 1 and 3 only
(c) 2-4-1-3
(d) 1-2-4-3 (d) 1, 2 and 3
13 www.visionias.in ©Vision IAS
67. With reference to coins in ancient India, 69. The Tropic of Cancer passes through this
which of the following statements are landlocked Indian state. Reang, Chakma, and
Jamatia are some of the major tribes found in
correct?
the state. The Thrishna Wildlife sanctuary is
1. The Saka coins of the Western situated in this state which hosts the Indian
Kshatrapas are one of the earliest dated subcontinent’s only ape species, the Hoolock
coins. Gibbon.
Which of the following states is best
2. Indo-Greeks were the first to issue gold
described in the passage given above?
coins in India.
(a) Madhya Pradesh
3. Lord Shiva and Buddha were portrayed (b) Manipur
on the coins of Kushana rulers. (c) Tripura
(d) Mizoram
4. Gupta rulers prohibited the use of gold
coins to protect the empire from
70. Consider the following statements with
economic decline. regard to the Anthrax disease:
Select the correct answer using the code given 1. Anthrax is a zoonotic disease.
2. It is caused by fungi.
below.
Which of the statements given above is/are
(a) 1 and 2 only
correct?
(b) 2 and 4 only (a) 1 only
(c) 1, 2 and 3 only (b) 2 only
(c) Both 1 and 2
(d) 1, 3 and 4 only
(d) Neither 1 nor 2

68. Consider the following statements with 71. With reference to the Indian Constitution,
respect to Harappan Script: consider the following statements with

1. Unlike the Egyptians and reference to the minister of tribal welfare of a


state:
Mesopotamians, the Harappans wrote
1. There is a provision in the Indian
long inscriptions. Constitution regarding the appointment
2. Most inscriptions were recorded on seals of a minister in charge of the tribal
and contain only a few words. welfare of a state.
2. All the states that have areas either under
3. It is not alphabetical but largely
the fifth schedule or sixth schedule are
pictographic. required to have a minister of tribal
Which of the statements given above is/are welfare.
correct? Which of the statements given above is/are
correct?
(a) 1 and 3 only
(a) 1 only
(b) 1 and 2 only
(b) 2 only
(c) 2 and 3 only (c) Both 1 and 2
(d) 3 only (d) Neither 1 nor 2
14 www.visionias.in ©Vision IAS
72. Which of the following statements is not 75. In the context of the food processing industry,
consider the following statements about the
correct regarding Additional Tier-1 (AT1)
Individual Quick Freezing Technique (IQF):
bonds? 1. Due to quick freezing, the original texture
(a) AT1 bonds are unsecured bonds issued and flavour of the food are maintained.
2. It does not allow large ice crystals to form
by banks.
in vegetable cells.
(b) These bonds are typically used by banks Which of the statements given above is/are
to bolster their tier-1 capital. correct?
(a) 1 only
(c) AT1 bonds, like other bonds, pay regular
(b) 2 only
interest to investors. (c) Both 1 and 2
(d) AT1 bonds are less riskier than vanilla (d) Neither 1 nor 2

corporate bonds.
76. With reference to the recently notified Green
Energy Open Access Rules, 2022, consider
73. Consider the following statements with the following statements:
1. The rules aim to incentivize the
regard to Chalukyan architecture:
consumers to use green power by
1. Huccimalligudi temple at Aihole is providing them green certificates for
rectangular in shape. using green power.
2. These rules were notified by the Ministry
2. The largest cave temple built by
of New & Renewable Energy.
Mangalesa is dedicated to Vishnu. Which of the statements given above is/are
3. Virupaksha temple has a Kannada correct?
(a) 1 only
inscription eulogizing the architect.
(b) 2 only
Which of the statements given above is/are (c) Both 1 and 2
correct? (d) Neither 1 nor 2

(a) 1 and 3 only


77. The RBI is concerned about slow deposit
(b) 2 and 3 only growth vis-a-vis credit growth. In this
(c) 2 only context, which of the following is/are the
possible reason(s) for slow deposit growth
(d) 1, 2 and 3
vis-a-vis credit growth?
1. High inflation in the economy
2. Increase in deposit rates by the banks
74. Recently country's first green Hydrogen
3. Higher returns on investments in capital
Mobility project has been decided to set up in market instruments
which of the following states/UTs? Select the correct answer using the code given
below.
(a) Gujarat
(a) 1 and 2 only
(b) Ladakh (b) 2 and 3 only
(c) Chandigarh (c) 3 only
(d) 1 and 3 only
(d) Mizoram
15 www.visionias.in ©Vision IAS
78. In the context of ecosystems in India, 80. Consider the following statements with
reference to the Animal Welfare Board of
consider the following statements regarding
India (AWBI):
Banni grasslands:
1. It is a statutory body established under the
1. They are located in Annamalai hills. Prevention of Cruelty to Animal Act,
1960 (PCA Act).
2. They are the habitat of stump-tailed
2. Its advice is binding on the Central and
macaques.
the state governments.
3. They are affected by the ingress of 3. It is headquartered in New Delhi.
Which of the statements given above is/are
Prosopis Juliflora, a harmful exotic tree
correct?
species.
(a) 1 only
Which of the statements given above is/are (b) 2 only
(c) 2 and 3 only
correct?
(d) 1, 2 and 3
(a) 1 only

(b) 1 and 2 only 81. Consider the following:


1. Blood tests
(c) 3 only
2. Pottery
(d) 2 and 3 only
3. Washing Machines
Which of the above uses centrifugal force to
function?
79. Recently, a sudden change in the colour and
(a) 1 and 2 only
smell of the crater lake Kuk caused anxiety
(b) 1 and 3 only
and panic among the local residents. It has (c) 1, 2 and 3
(d) 2 only
been observed that lakes of this region emit

lethal amounts of gases, such as carbon


82. Consider the following statements regarding
dioxide, from the Earth’s mantle. These gases the first Anglo-Sikh War:
1. The immediate trigger for the war was
have been accumulating at the bottom of the
Sikh army crossing the river Sutlej.
lakes of this region for centuries and cause
2. The war ended with the Treaty of
fatalities. Gandamak.
3. Henry Hardinge was the Governor
The above passage refers to lakes of which of
General at the time of the war.
the following countries?
Which of the statements given above is/are
(a) United States correct?
(a) 1 only
(b) Cameroon
(b) 2 and 3 only
(c) Russia
(c) 1 and 3 only
(d) Turkey (d) 1, 2 and 3
16 www.visionias.in ©Vision IAS
83. Fungi possess the biochemical and ecological 86. Consider the following statements regarding
capacity to degrade environmental organic the different forms of puppetry in India:
chemicals and as such are used in 1. The string puppets of Karnataka are
bioremediation. In this context, which of the called Tholu Bommalata.
following fungi species is/are used in 2. The Ravanachhaya puppets of Odisha are
bioremediation? in one piece and have no joints.
1. Pleurotus 3. The traditional rod puppet of Bihar is
2. Aspergillus known as Putul Nach.
3. Trichoderma Which of the statements given above is/are
Select the correct answer using the code given correct?
below. (a) 1 and 3 only
(a) 1 only (b) 2 and 3 only
(b) 1 and 3 only (c) 2 only
(c) 2 and 3 only (d) 1 and 2 only
(d) 1, 2 and 3
87. Which of the following statements is/are
84. In the context of astronomy, which of the correct with reference to the Chief Justice of
following statements best describes the term India?
“Oort Cloud”? 1. He is the patron-in-chief of the National
(a) It is the region of icy bodies between Legal Services Authority (NALSA).
Mars and Jupiter. 2. He can use veto power as a member of the
(b) It is the vivid clouds in Neptune's collegium in case of a difference of
atmosphere. opinion.
(c) It is the most distant region of our solar 3. He can be removed by the Parliament of
system consisting of icy pieces of space India on the grounds of proven
debris. misbehavior and incapacity.
(d) It is a giant cloud of dust and gas in space Select the correct answer using the code given
which can cause the birth of stars. below.
(a) 1 and 2 only
85. With reference to the Indian Councils Act of (b) 1 only
1909, consider the following statements: (c) 1, 2 and 3
1. It retained an official majority in the (d) 2 and 3 only
Central Legislative Council.
2. It provided for the association of Indians 88. Which of the following statements is not
with the executive councils of the correct regarding Montreal Protocol?
Viceroy and Governors. (a) It is a non-binding multilateral agreement
3. It provided for the separate representation that regulates the production and use of
of chambers of commerce, universities, ozone depleting substances.
and zamindars. (b) Under this protocol, both the developing
Which of the statements given above is/are and developed countries have equal but
correct? differentiated responsibilities.
(a) 1 only (c) The Protocol promotes technology
(b) 1 and 2 only transfer to developing countries.
(c) 2 and 3 only (d) The Protocol has achieved universal
(d) 1, 2 and 3 ratification.
17 www.visionias.in ©Vision IAS
89. Consider the following statements regarding 92. Which of the following best describes the
UNSC 1267 Committee: phenomenon of ‘Avulsions’ in the river
1. It allows a UN member state to propose course?

adding the name of a terrorist or terror (a) A feature formed by streams flowing

group to a consolidated list, maintained from higher levels breaks into foot slope
plains.
by the Committee.
(b) Natural diversion of the headwaters of
2. It was set up after the 9/11 twin towers
one stream into the channel of another.
attack on the USA.
(c) The rapid abandonment of a river channel
Which of the statements given above is/are
and the formation of a new river channel.
correct?
(d) An erosion process resulting from
(a) 1 only
lowering of the base level of the river.
(b) 2 only
(c) Both 1 and 2 93. Consider the following statements regarding
(d) Neither 1 nor 2 follow-on-public offer (FPO) and initial
public offer (IPO):
90. The terms ‘ChatGPT, DALL-E and LaMDA’ 1. Unlike in an IPO, the company that
sometimes mentioned in the news recently releases an FPO is already listed on a
are related to stock exchange.

(a) Artificial Intelligence 2. Generally, investing in an IPO is

(b) Crypto-currency considered less risky than investing in an


FPO.
(c) Nano-satellites
Which of the statements given above is/are
(d) Payment Gateways
correct?
(a) 1 only
91. Consider the following statements with
(b) 2 only
reference to the Contingency Fund of India:
(c) Both 1 and 2
1. As per the Constitution of India, the
(d) Neither 1 nor 2
President of India is authorized to
establish a Contingency Fund of India. 94. Which of the following best describes the
2. The fund is in the nature of an imprest (a word 'Chillai Kalan'?
fixed fund for a specific purpose) (a) Diurnal temperature variation in the
account. region of Thar desert.
Which of the statements given above is/are (b) Spell of cold wave in the region of
correct? Kashmir.

(a) 1 only (c) Sharp decrease in humidity in the region

(b) 2 only of Jharkhand.


(d) Local name of winter rainfall in Punjab
(c) Both 1 and 2
plains.
(d) Neither 1 nor 2
18 www.visionias.in ©Vision IAS
95. Which of the following reasonable 98. UNFCCC termed three critical problems
restrictions on the Right to Freedom of faced by Earth as " Triple Planetary Crisis ".
Residence (Article 19) has/have been These problems are:
explicitly mentioned in the Indian (a) Climate Change, Sea level rise, Land
Constitution? degradation
(b) Pollution, Land degradation, Sea level
1. Security of the state
rise
2. Sovereignty and integrity of India
(c) Climate change, Pollution, Biodiversity
3. Morality
loss
4. The protection of the interests of any (d) Pollution, Biodiversity loss, Land
scheduled tribes degradation
Select the correct answer using the code given
below. 99. He was born into a family of industrialists. He
(a) 1, 2 and 3 only shifted to the University of Cambridge,
(b) 1 and 4 only England after college. He was appointed
(c) 2, 3 and 4 only chairman of the Atomic Energy Commission
(d) 4 only of India. He laid the foundations for the
indigenous development of nuclear
technology for defence purposes. He was
96. Recently over 200 prehistoric paintings were
awarded two of India’s highest honours, the
identified at a cave in the forest area of Bargur
Padma Bhushan and the Padma Vibhushan.
hills. This cave is located in which of the
Who among the following is being described
following states? in the above-given passage?
(a) Andhra Pradesh (a) Homi J Bhabha
(b) Karnataka (b) Vikram A Sarabhai
(c) Kerala (c) C.V. Raman
(d) Tamil Nadu (d) A.P.J. Abdul Kalam

97. Recently, the Reserve Bank of India released 100. 18th century India saw the rise of literary
its report on state government budgets for works in regional languages. In this context,
2022-23. In this context, consider the consider the following pairs:
Poets Composed in the
following statements:
language of
1. The debt-to-GDP ratio of states remains
1. Warris Shah : Sindhi
high over 40 percent.
2. Shah Abdul Latif : Urdu
2. The fiscal health of the states has
3. Tayumanavar : Kannada
improved with a decline in the gross
4. Kanchan : Malayalam
fiscal deficit.
Nambiar
Which of the statements given above is/are
Which of the pairs given above is/are
correct?
correctly matched?
(a) 1 only
(a) 1 and 3 only
(b) 2 only (b) 2 and 4 only
(c) Both 1 and 2 (c) 4 only
(d) Neither 1 nor 2 (d) 1 and 2 only

Copyright © by Vision IAS


All rights are reserved. No part of this document may be reproduced, stored in a retrieval system or transmitted
in any form or by any means, electronic, mechanical, photocopying, recording or otherwise, without prior
permission of Vision IAS.
19 www.visionias.in ©Vision IAS
VISIONIAS
www.visionias.in
ANSWERS & EXPLANATIONS
GENERAL STUDIES (P) TEST – 3808 (2023)

Q 1.C
• The freedom guaranteed by Article 301 is freedom from all restrictions, except those which are provided
for in the other provisions (Articles 302 to 305) of Part XIII of the Constitution itself. These are explained
below:
o Parliament can impose restrictions on the freedom of trade, commerce, and intercourse between
the states or within a state in the public interest. But, the Parliament cannot give preference to one
state over another or discriminate between the state except in the case of scarcity of goods in any part
of India.
o The legislature of a state can impose reasonable restrictions on the freedom of trade, commerce,
and intercourse with that state or within that state in the public interest. But, a bill for this purpose
can be introduced in the legislature only with the previous sanction of the president. Further, the
state legislature cannot give preference to one state over another or discriminate between the states.
• Hence option (c) is the correct answer.

Q 2.D
• The Constitution (Seventy-third Amendment) Act, 1992 empowered local governments – rural, as
constitutional entities and initiated a new chapter in the process of democratic decentralization in
India. This act has added a new Part-IX to the Constitution of India. This part is entitled ‘The Panchayats’
and consists of provisions from Articles 243 to 243 O. In addition, the act has also added a new Eleventh
Schedule to the Constitution. This schedule contains 29 functional items of the panchayats.
• The act has given a practical shape to Article 40 of the Constitution which says that, “The State shall take
steps to organize village panchayats and endow them with such powers and authority as may be necessary
to enable them to function as units of self-government.” This article forms a part of the Directive Principles
of State Policy.
• The provisions of the act can be grouped into two categories– compulsory and voluntary. The compulsory
(mandatory or obligatory) provisions of the act have to be included in the state laws creating the new
Panchayati raj system.
o The 73rd Amendment Act, 1992 clearly states that no person shall be disqualified on the ground
that he is less than twenty-five years of age if he has attained the age of twenty-one years. Thus 21
years has been set as the minimum age for the election of a person to the Panchayat. Hence statement
1 is not correct.
o Further, all questions of disqualifications shall be referred to such authority as the state legislature
determines (and not strictly the high court). Hence statement 2 is not correct.
Q 3.B
• Death penalty or capital punishment can be defined as a practice sanctioned by law whereby a person is put
to death by the state as a punishment for a crime after a proper legal trial’.
• India is among the few countries that retain capital punishment under different laws for serious offences-
beyond the Law Commission recommendations.
• In 1980, in Bachan Singh vs. State of Punjab, SC Judges upheld the constitutional validity of the death
penalty due to built-in reasonable procedural safeguards and its procedure which is neither arbitrary nor
gives excessive discretion to judges. Hence statement 1 is correct.
• As per law, no death sentence imposed by a trial court can be executed unless the punishment is confirmed
by the High Court too and the Code of Criminal Procedure (CrPC) of 1973 requires trial courts to refer
every judgment related to the imposition of capital punishment to the jurisdictional High Court for
confirmation.
1 www.visionias.in ©Vision IAS
• In 1975, three persons sentenced to death for the same crime ended up suffering different punishments
because separate legal processes had been followed for each convict. Shocked by the inconsistency that
resulted in one convict being hanged while another had his sentence commuted, and in order to ensure that
this does not happen again, the Supreme Court directed that convicts in the same crime must be executed
together.
• For a person convicted by courts, a mercy plea is the last constitutional resort, provided under Article 72
(President), and Article 161 (Governor). Neither the term 'mercy petition' has been mentioned in Article
72 nor explained anywhere in the Indian Constitution. There is no statutory written procedure for
dealing with mercy petitions, but in practice, after extinguishing all the reliefs in the court of law,
either the convict in person or his relative on his behalf may submit a written petition to the President.
The petitions are received by the President’s secretariat on behalf of the President, which is then forwarded
to the Ministry of Home Affairs for their comments and recommendations. Hence statement 2 is not
correct.
• Issues with Mercy Petition:
o No fixed timeframe to act on the mercy plea leading to long delays.
o Law Commission has highlighted certain Presidents who put brakes on the disposal of the Mercy
Petition.
o Lack of transparency as there is no compulsion to share reasons for rejection or acceptance of a mercy
plea, but it is subjected to Limited Judicial Review (Epuru Sudhakar & Anr. v. Government of Andhra
Pradesh case, 2006).

Q 4.A
• pH change as the cause of tooth decay Tooth decay starts when the pH of the mouth is lower than 5.5.
• Tooth enamel, made up of calcium hydroxyapatite (a crystalline form of calcium phosphate) is the
hardest substance in the body. Hence, option (a) is the correct answer.
• It does not dissolve in water but is corroded when the pH in the mouth is below 5.5.
• Bacteria present in the mouth produce acids by degradation of sugar and food particles remaining in the
mouth after eating.
• The best way to prevent this is to clean the mouth after eating food.
• Using toothpaste, which is generally basic, for cleaning the teeth can neutralise the excess acid and prevent
tooth decay.

Q 5.A
• Chimeric antigen receptor (CAR) T-cell therapy is a way to get immune cells called T cells (a type of
white blood cell) to fight cancer by changing them in the lab so they can find and destroy cancer cells.
• While CAR-T cell therapy has seen success in blood cancers, it has faced hurdles when fighting what are
called solid tumor cancers like pancreatic cancer and melanoma.
• Unlike cancers that begin in the blood, these types of cancers grow into a solid mass that produces a
microenvironment of molecules, cells and structures that prevent T cells from entering into the tumor and
triggering an immune response. Here, even CAR-T cells engineered to specifically target a patient’s unique
tumor are unable to access it, suppressing their ability to kill tumor cells.
• In 2016, researchers in Lim Lab at the University of California, San Francisco developed a new
synthetic receptor that could complement the first CAR design. This receptor, called synthetic Notch
receptor, or synNotch, is based on the natural form of Notch in the body, which plays an important role in
organ development across many species.
• Similar to CARs, the outside of synNotch has a binder that allows T cells to stick to tumor cells. Unlike
CARs, the inside of synNotch has a protein that is released when a T cell binds to the tumor. This protein,
or transcription factor, allows researchers to better control the T cell by inducing it to produce a specific
protein.
• Hence, option (a) is the correct answer.

Q 6.B
• A trend in the percentage share of agriculture and allied sectors to the total GVA of the economy at current
prices for the last ten years is presented below. The share of the sector in total GVA of the economy has
a long-term trend of around 18 per cent. The share of the agriculture & allied sector in total GVA,
however, improved to 20.2 per cent in the year 2020-21 and 18.8 per cent in 2021-22. Hence statement 1
is not correct.
• Gross Capital Formation (GCF) in agriculture and allied sectors relative to GVA in this sector has been
showing a fluctuating trend. Fluctuation in the GCF in the sector arises mainly because of wide fluctuations
2 www.visionias.in ©Vision IAS
in private investment in agriculture and allied sectors. Most private investments in agriculture—be it in
irrigation, mechanisation or land development—is undertaken by farmers. As regards capital
formation in agriculture in India, about 76% is from private sector (with farm household investment
forming over 70%) and 24% from public sector. Hence statement 2 is correct.
• Profile of the Agriculture Sector in India
o In 5 states viz. Andhra Pradesh, Arunachal Pradesh, Madhya Pradesh, Nagaland and Tripura,
Agriculture and Allied activities contribute more than 30% in State Gross Value Added estimated for
2016-17 (at current prices).
o The number of operational holdings in the country is estimated at 14.64 Crore. (Agriculture Census
2015-16).
o The percentage share of Agricultural workers in Total workers is 54.6%. (Registrar General of India)
o 57.8% of rural households are engaged in agriculture (Situation Assessment Survey of
Agricultural Households, NSO). Hence statement 3 is not correct.
o The small and marginal holdings taken together (0.00-2.00 ha) constituted 86.08% of the total land
holdings in 2015-16. The all- India average size of holding is 1.08 ha. (Source: Agriculture Census,
2015-16)
o Position of women in agriculture
✓ 30.33% of total cultivators and 40.67% of agriculture labour are women.
✓ Only 13.95% of total operational holdings are operated by women. (Source: Agriculture Census,
2015-16)

Q 7.C
• Mhadei, also known as the Mandovi river in Goa and Mahadayi in Karnataka, is considered a lifeline in the
northern parts of Goa. Mahadayi river rises in the Western Ghats, from the Bhimgad Wildlife
Sanctuary in Khanapur taluk of Karnataka’s Belagavi district. Flowing westward, it enters Goa from
the Sattari taluk of North Goa districts. A number of streams join the flow of the river to form the Mandovi
which is one of two major rivers that flow through Goa. It joins the Arabian Sea at Panaji. Hence, statement
1 is correct.

• Currently, the states of Goa and Karnataka are in a dispute due to the decision of the state of Karnataka
to route water from the Mhadei river to the Malaprabha basin. The dispute is lingering since the 1980s.
Karnataka claims that the surplus from Mahadayi drains into the sea and that it should be diverted into the
deficit basin in Malaprabha to meet the state’s drinking, irrigation, agriculture and power generation needs.

3 www.visionias.in ©Vision IAS


Goa has, meanwhile, denied Karnataka’s claims saying it is a water-deficient state and limiting the water
supply would adversely impact its agriculture production. The Mhadei river drains into the Arabian Sea
and is not a tributary of the Malaprabha river. Hence, statement 2 is not correct.
o The Malaprabha River is a tributary of the Krishna River and flows through the state of Karnataka
in India. It rises in the Western Ghats in the Belgaum district of Karnataka.
• Dudhsagar waterfall is a four-tiered waterfall located on the Mandovi River(Mhadei) in Goa. It lies
at a distance of 60 km from Panaji and 10 km from Kulem. The falls are located in the Bhagwan
Mahaveer Sanctuary and Mollem National Park among the Western Ghats. Hence, statement 3 is
correct.

Q 8.C
• ‘Dharamshala Declaration 2022’ was released in September 2022 at a three-day national conference on
tourism.
• The Dharamshala Declaration aims to recover tourism to the pre-pandemic level by 2024 and position
India as a major tourism destination during its presidency of G-20 next year. Hence statement 1 is correct.
• It affirms commitment toward developing “sustainable and responsible tourism” and positions India as
a “global leader in the tourism sector by 2047”. In the medium term (that is 2030), tourism-related goals
are USD 250 billion GDP contribution; 137 million jobs and 56 million foreign tourist arrivals. In the long
term, it aims to revive India’s tourism and targets to achieve USD 1 Trillion by the sector in 2047.
Hence statement 2 is correct.

Q 9.D
• Precision farming is an approach where inputs are utilised in precise amounts to get increased
average yields, compared to traditional cultivation techniques.
• An information and technology-based farm management system identifies, analyses, and manages
variability in fields by conducting crop production practices at the right place and time and in the right way,
for optimum profitability, sustainability, and protection of the land resource.
• Technologies include a vast array of tools of hardware, software, and equipment:
o Global Navigation Satellite Systems (GNSS) receivers
o Differential Global Positioning System (DGPS)
o Geographic Information Systems (GIS)
o Remote Sensing (RS) technique
o Variable Rate Applicator
o Combine harvesters with yield monitors
• Hence option (d) is the correct answer.
• Sustainable precision farming and healthy food production increase profitability and output, improve
economic efficiency, and lessens adverse environmental effects. Other advantages include efficient use of
water resources, prevention of soil degradation and even improving farmers' socio-economic standing.
• Precision farming also enables climate-smart agri-business which is necessary for achieving climate goals.

Q 10.D
• Net metering is a policy of Government of India that allows consumers with solar panels or other forms of
renewable energy generation to feed excess electricity back into the grid and receive credits on their
utility bill. This policy is intended to promote the use of renewable energy and help consumers offset the
cost of their solar panels or other equipment. Hence pair 1 is correctly matched.
• KUSUM (Kisan Urja Suraksha evam Utthaan Mahabhiyan) is a scheme launched by the Government
of India to promote the use of solar power by farmers. The scheme aims to provide financial assistance to
farmers to install solar power systems on their land and also to encourage them to generate income by
selling surplus power to the grid. The scheme includes various components, such as the installation of
grid-connected solar power plants, off-grid solar water pumps, and solarized agriculture pumps. The
scheme is intended to empower farmers to become self-sufficient in meeting their energy needs, and to help
reduce the country's dependence on fossil fuels. Hence pair 2 is correctly matched.
• SRISTI is Sustainable Rooftop Implementation for Solar Transfiguration of India. It is a type of
scheme that will offer with financial incentive to the beneficiary for installing Solar power plant
rooftop projects within the country. The incentives will be offered by the central government. Hence
pair 3 is correctly matched.
• Atal Jyoti Yojana (Atal Jyoti Scheme) is a scheme launched by the government of India. The scheme
aims to improve the power supply situation in rural areas and reduce transmission and distribution losses
through the use of smart metering and energy-efficient technologies. It also includes the installation of
4 www.visionias.in ©Vision IAS
solar-powered street lights and provision of power supply to un-electrified villages, and to households
that are yet to be connected to the grid. Hence pair 4 is correctly matched.

Q 11.A
• Phytoplankton, also known as microalgae, are similar to terrestrial plants in that they contain
chlorophyll and require sunlight in order to live and grow. Most phytoplankton are buoyant and float in the
upper part of the ocean, where sunlight penetrates the water.
o Phytoplankton also require inorganic nutrients such as nitrates, phosphates, and sulfur which they
convert into proteins, fats, and carbohydrates.
• The two main classes of phytoplankton are dinoflagellates and diatoms. Dinoflagellates use a whip-like
tail, or flagella, to move through the water and their bodies are covered with complex shells. Diatoms also
have shells, but they are made of a different substance and their structure is rigid and made of interlocking
parts.
o Diatoms do not rely on flagella to move through the water and instead rely on ocean currents to travel
through the water.
• Coccolithophores are also a group of phytoplankton that live in large numbers throughout the upper
layers of the ocean. Coccolithophores surround themselves with a microscopic plating made of limestone
(calcite).
• Crustaceans include all the animals of the phylum Arthropoda. Crustaceans are a very diverse group
of invertebrate animals that includes active animals such as crabs, lobsters, shrimp, krill, copepods,
amphipods, and more sessile creatures like barnacles. Arthropoda is the largest phylum of Animal Kingdom.
• Foraminifera represents an ancient species of zooplankton that live mostly in sediment but also in the
water column. They belong to the kingdom Protista.
• Phytoplankton varies seasonally in amount, increasing in spring and fall with favourable light, temperature,
and minerals.
• Hence option (a) is the correct answer.

Q 12.D
• The Second Schedule of the Indian Constitution contains the provisions relating to the emoluments,
allowances, and privileges for different offices. The various offices are:
o The President of India
o The Governors of States
o The Speaker and the Deputy Speaker of the Lok Sabha
o The Chairman and the Deputy Chairman of the Rajya Sabha
o The Speaker and the Deputy Speaker of the Legislative Assembly in the states
o The Chairman and the Deputy Chairman of the Legislative Council in the states
o The Judges of the Supreme Court
o The Judges of the High Courts
o The Comptroller and Auditor General of India.
• The provisions relating to the emoluments, allowances, and privileges of state chief ministers and
council of ministers are not mentioned in the Second Schedule. Hence option (d) is the correct answer.
• The provisions in the Second Schedule can be amended by a simple majority of the two Houses of
Parliament outside the scope of Article 368.
Q 13.D
• The Investor Education and Protection Fund Authority (IEPFA) was set up in 2016, under the aegis of the
Ministry of Corporate Affairs, Government of India, for administering the Investor Education and
Protection Fund.
• Investor Education and Protection Fund Authority was established under the provisions of section 125 of
the Companies Act, 2013. Hence option (d) is the correct answer.
• The IEPFA Authority is entrusted with the responsibility of administration of the Investor Education
Protection Fund (IEPF), making refunds of shares, unclaimed dividends, matured deposits/debentures etc.
to investors, promoting awareness among investors, and protecting the interests of the investors.
• Investor Education and Protection Fund is utilized for
o The refund in respect of unclaimed dividends, matured deposits, matured debentures, the application
money due for refund and interest thereon
o Promotion of investors’ education, awareness and protection
o Distribution of any disgorged amount among eligible and identifiable applicants for shares or
debentures, shareholders, debenture-holders or depositors who have suffered losses due to wrong
actions by any person, in accordance with the orders made by the Court which had ordered disgorgement
5 www.visionias.in ©Vision IAS
Q 14.A
• The Parliament is too unwieldy a body to deliberate effectively on the issues that come up before it. The
functions of the Parliament are varied, complex, and voluminous. Moreover, it has neither the adequate time
nor the necessary expertise to make a detailed scrutiny of all legislative measures and other matters.
Therefore, it is assisted by a number of committees in the discharge of its duties.
• The Constitution of India makes a mention of these committees at different places, but without
making any specific provisions regarding their composition, tenure, functions, etc. All these matters
are dealt with by the rules of two Houses. Accordingly, a parliamentary committee means a committee
that: Hence statement 1 is not correct.
o Is appointed or elected by the House or nominated by the Speaker / Chairman.
o Works under the direction of the Speaker / Chairman.
o Presents its report to the House or to the Speaker / Chairman.
o Has a secretariat provided by the Lok Sabha / Rajya Sabha.
• Broadly, parliamentary committees are of two kinds-Standing Committees and Ad Hoc Committees.
o The standing committees are permanent (constituted every year or periodically) and work on a
continuous basis. Hence statement 2 is correct.
o The Ad Hoc Committees are temporary and cease to exist on completion of the task assigned to
them. Ad hoc committees can be divided into two categories, that is, Inquiry Committees and
Advisory Committees.
✓ Inquiry Committees are constituted from time to time, either by the two Houses on a motion
adopted in that behalf or by the Speaker / Chairman, to inquire into and report on specific subjects.
For example, the committee on the Conduct of Certain Members during President’s Address.
✓ Advisory Committees include select or joint committees on bills, which are appointed to consider
and report on particular bills. These committees are distinguishable from the other ad hoc
committees in as much as they are concerned with bills and the procedure to be followed by them
is laid down in the Rules of Procedure and the Directions by the Speaker / Chairman.

Q 15.A
Foreign Direct Investment (FDI) Policy in Agriculture Sector:
• As per the extant FDI Policy, 100% FDI is allowed in the following sector through an automatic route:
o Floriculture, Horticulture, Cultivation of Vegetables & Mushrooms under controlled conditions;
o Development and Production of seeds and planting material;
o Animal Husbandry (including breeding of dogs), Pisciculture, Aquaculture, Apiculture and
o Services related to agro and allied sectors
• Further, 100% FDI is also permitted in tea sector including tea plantations, Coffee plantations, Rubber
plantations, Cardamom plantations, Palm Oil plantations and Olive oil tree plantations through automatic
route.
• FDI in Multi-Brand retail is also permitted up to 51% through the Government route in the agricultural
sector with some conditions.
• Hence option (a) is the correct answer.

Q 16.C
• The rise of the Sarvāstivāda school of Buddhism as a distinct group dates back to the 2nd to 1st centuries
BCE. They attained prominence in the region of Mathura and in Kashmir.
• According to Buddhist historical literature, the Sarvāstivādins split from the Sthaviravāda school at the
third Buddhist council held in Pāṭaliputra (present-day Patna).
o The third council led the divergence of Sthaviravāda into two schools: Vibhajyavādins and
Sarvāstivādins. Then, a new school of Sarvāstivādins, Sautrāntikas spring off.
• The name “Sarvāstivāda” suggests that the disagreement with the Sthaviravādins was a matter of doctrinal
viewpoint: “sarvāstivāda” is derived from the Sanskrit sarvam asti, meaning essentially “everything
exists/all is.” The question from which the school derives its name is whether discrete entities (dharma)
have an existence only in the present, or whether they also exist as such in the past and future. In other
words, the question is whether the past becomes manifest in the present, and whether the future is already
latent in the present. This focus on philosophical interpretation and debate explains why the
Sarvāstivāda school predominantly is an abhidharma school.
• The Fourth Buddhist Council was held in Kashmir by the patronage of Kanishka in Kashmir about
72-78 AD. It was presided over by Vasumitra and Asvaghosha and had to deal with a serious conflict
between the Sarvasthivada teachers of Kashmir and Gandhara.
6 www.visionias.in ©Vision IAS
o During this meeting the Sarvasthavadin doctrines were organized into a Mahavibhasa containing
three large commentaries on the Pitakas. They were codified in a summary. Hence option (c) is
the correct answer.
o In the fourth council, the Sarvāstivādins were dominant. They debated on the orthodoxy and the
authenticity of their teachings at this council.
o One was the Sarvāstivāda and the Sautrāntika. The Sautrāntika was the school that began to be critical
of the realism and pluralism of the Sarvāstivāda. The Sarvāstivāda regarded them as real while the
Sautrāntika regarded them as mentally created. The meaning of the name Sautrāntika comes from the
term ’Sūtra’, that is, those who adhere to Sūtra.
o There is a principal difference noticed between the Sautrāntika and the Sarvāstivāda. The Sarvāstivāda
was also called Vaibhāṣika, the followers of the Vibhāṣā or Commentaries. The Sarvāstivādins believed
that the Abhidharma was the word of the Buddha whereas the Sautrāntikas did not accept that.
o At this council, the Vaibhāṣikas, the Sarvāstivādins won the debate and the Sautrāntikas were
disgraced.

Q 17.A
• India's coal consumption has doubled since 2007 at an annual growth rate of 6 per cent. Coal production in
2021 reached 800 million tonnes for the first time and is forecast to surpass 1 billion tonnes by 2025.
• India is the world's second-largest producer and consumer of coal. Three-quarters of the electricity
produced in the country is through coal. India sits atop the world's third-highest reserves of dry fuel and
boasts of the world's largest coal mining company. Hence statement 1 is correct.
• Coal India Limited, the largest supplier of coal in the country contributing more than 80% of the
indigenous production/supply. It has envisaged a plan to enhance its production to reach the level of one
Billion Tonne (BT) coal by the year 2024-25 from its current production level of about 622 MT in order to
meet the demand of coal indigenously and to eliminate non-essential import of coal in the country. Hence
statement 2 is correct.
• ICI measures combined and individual performance of production in selected eight core industries viz. Coal,
Crude Oil, Natural Gas, Refinery Products, Fertilizers, Steel, Cement and Electricity. It is released by The
Office of Economic Adviser, Department for Promotion of Industry and Internal Trade (DPIIT). Coal
production has a weightage of 10.33% in the Index of Core Industries (ICI). Hence statement 3 is not
correct.

Q 18.D
• Recently, Sri Lanka has proposed that the Indian government can assist by providing six specimens of the
gaur — a bull, and three to five cows — for captive breeding over a five-year period.
• The Gaur or Indian Bison is a species of wild cattle native to Asia and South East Asia. Thus it is not
endemic to India. Hence statement 1 is not correct.
• The Indian Bison is massively built animal and is very strong. They have a convex shape in the forehead.
The limbs are very strong and sturdy. The adult male weighs around 600 kg to 1500 kg. and the adult female
weighs about 400 kg to 1000 kg.
• The Gaur is a social animal. They generally live in group size of about 30 to 40.
• The Indian Bison is very much prevalent in the Western Ghats. They prefer evergreen forests and moist
deciduous forests. However, they can survive in dry deciduous forests also. They are not found in the
Himalayas with an altitude greater than 6,000 ft. They generally stick to the foothills only.
• The Indian Bison is deemed as vulnerable according to the IUCN list. Hence, the Indian Government
has already included the protection of wild bison in the Schedule I of the Wild Life Protection Act,
1972. Hence statement 2 is not correct.
• In India Gaur can be found in national parks like Nagarhole National Park, Bandipur National
Park, Periyar National Park, Mudumalai National Park, Kaziranga National Park etc. Hence statement 3
is correct.

Q 19.D
• Despite the Central government’s push towards capital expenditure (capex), the total public sector capex is
likely to decline as a percentage of GDP in 2022-23 (FY23), as state governments and public sector
enterprises are unlikely to meet their capex targets. Public sector capex is likely to fall to an eight-year low
of 5.7% of GDP in FY23.
• The Centre’s capex had spiked to 2.3% of GDP last year, far higher than pre-pandemic levels of 1.5-1.9%
and has risen by another 52% in the first half of this year to touch ₹3.2 lakh crore or 2.5% of GDP. Hence
statement 1 is not correct.
7 www.visionias.in ©Vision IAS
• The sharp surge in the Centre’s capex over the past 18 months or so does not imply higher public sector
capex. This is mainly because of a drop in investments by central public sector enterprises (CPSEs).
While the Centre’s capex is budgeted to rise in 2022-23, it is likely to contract for the third consecutive
year for CPSEs. Hence statement 2 is not correct.
• Additionally, State capex is likely to remain stable and is unlikely to achieve the budget estimate of ₹6.9
lakh crore, which would have accounted for 2.6% of GDP. Instead, even with the expected ramp-up in
capex in H2 FY23, states will likely only hit ₹5.9 lakh crore in capex, which would be 2.1% of GDP, a
similar level to that seen in the past few years.

Q 20.D
• Pyrolysis is the heating of an organic material, such as biomass, in the absence of oxygen. It is considered
as one of the most favorable and effective disposing of solid waste methods, which is an environmentally
friendly and efficient way. Hence statement 1 is not correct.
• Pyrolysis of carbonaceous wastes like firewood, coconut, palm waste, corn combs, cashew shell, rice
husk paddy straw and sawdust, yields charcoal along with products like tar, methyl alcohol, acetic
acid, acetone and fuel gas. Hence statement 2 is correct.
• The key difference between incineration and pyrolysis is that incineration is the combustion of organic
matter in the presence of oxygen whereas pyrolysis is the combustion of organic matter in the absence of
oxygen.
o Issues with Incineration
✓ It reduces the recycling of recyclable materials because burning is easier and cheaper than recycling.
✓ It releases gaseous components such as carbon dioxide (CO2).
✓ It emits hazardous end products.
✓ It can lead to different health risks.
• Pyrolysis is an extremely efficient ways of using biomass to produce energy, both being more efficient
than incineration. These technologies are cleaner than incineration and do not pose toxicity threats.
Incineration also produces a highly toxic fly ash that must be safely disposed of; leading to transportation
and residential health concerns. Pyrolysis on the other hand is a cleaner processes and do not pose toxicity
threats, but remain “third generation” technologies. Hence statement 3 is correct.

Q 21.C
• The Law Commission of India is a non-statutory body and is constituted by a notification of the Government
of India, Ministry of Law & Justice, Department of Legal Affairs with a definite term of reference to carry
out research in the field of law and the Commission make recommendations to the Government (in the form
of Reports) as per its terms of reference.
• The Law Commission has taken up various subjects on references made by the Department of Legal Affairs,
Supreme Court, and High Courts and submitted 277 reports. The Law Commission of India provides an
excellent thought-provoking and vital review of the laws in India. Hence statement 1 is not correct.
• The first Law Commission was established during colonial rule in India by the East India Company
under the Charter Act of 1833 and was presided over by Lord Macaulay. After that, three more
commissions were established in pre-independent India.
• The Central Government established the First Law Commission in 1955 with the then Attorney-
General of India, Mr. M. C. Setalvad, as its Chairman. Hence statement 2 is correct.
• Since then twenty-two Law Commissions have been appointed, each with a three-year term and with a
definite term of reference. After two and a half years, the 22nd Law Commission of India has been
constituted. The Centre on November 7, 2022, constituted the Law Commission of India with Justice (retd)
Rituraj Awasthi, former Chief Justice of Karnataka High Court, at its head. The tenure of the 21st Law
Commission, which was headed by former Supreme Court judge Justice B S Chauhan, came to an end on
August 31, 2018. The 22nd Commission has been constituted two and a half years after it was
approved by the Union Cabinet on February 19, 2020, just before the outbreak of the Covid-19
pandemic. A petition had been moved in the Supreme Court subsequently against the delay in
constituting the 22nd Commission. Hence statement 3 is not correct.

Q 22.C
• A significant feature of the Quit India Movement was the emergence of what came to be known as
parallel governments in some parts of the country. Among different places, the parallel governments in
Satara in Maharashtra, Tamluk in the Midnapur district of Bengal and Ballia, in East United Province were
most prominent.

8 www.visionias.in ©Vision IAS


• The parallel government Satara lasted for the longest duration. It started in August 1943 and
remained till May 1945. Here the parallel government came to be known as Prati Sarkar. The leaders
like Y.B. Chauhan and Nana Patil provided leadership to the parallel government. The activities of the
parallel government were marked by attacks on Government collaborators, informers and talatis or lower-
level officials and Robin Hood-style robberies.
• Nyayadan Mandals or people’s courts were set up and justice dispensed. Hence statement 3 is correct.
• Prohibition was enforced, and ‘Gandhi marriages’ celebrated to which untouchables were invited and at
which no ostentation was allowed. Village libraries were set up and education encouraged. Hence,
statement 4 is correct.
• The native state of Aundh, whose ruler was pro-nationalist and had got the constitution of his state drafted
by Gandhiji, provided invaluable support by offering refuge and shelter to the Prati Sarkar
activists. The Prati Sarkar continued to function till 1945.
• The first parallel government during the Quit India Movement was proclaimed in Ballia, in East U
P, in August 1942 under the leadership of Chittu Pande. Hence statement 1 is not correct.
• In Tamluk in the Midnapur district of Bengal, the Jatiya Sarkar came into existence on 17 December, 1942
and lasted till September 1944. The Jatiya Sarkar also established Vidyut Vahini, an armed
organisation to carry out revolutionary activity. Hence, statement 2 is not correct.

Q 23.B
• What Is an Inflationary Gap?
o An inflationary gap is a macroeconomic concept that measures the difference between the current
level of real gross domestic product (GDP) and the GDP that would exist if an economy was
operating at full employment. Hence, statement 1 is not correct.
o KEY TAKEAWAYS
✓ An inflationary gap measures the difference between the current level of real GDP and the GDP
that would exist if an economy was operating at full employment.
✓ For the gap to be considered inflationary, the current real GDP must be higher than the
potential GDP. Hence, statement 2 is not correct.
✓ Policies that can reduce an inflationary gap include reductions in government spending, tax
increases, bond and securities issues, interest rate increases, and transfer payment reductions.
Hence, statement 3 is correct.
o An inflationary gap exists when the demand for goods and services exceeds production due to factors
such as higher levels of overall employment, increased trade activities, or elevated government
expenditure.
o Against this backdrop, the real GDP can exceed the potential GDP, resulting in an inflationary gap. The
inflationary gap is named as such because the relative rise in real GDP causes an economy to increase
its consumption, leading prices to climb in the long run.
o The inflationary gap represents the point in the business cycle when the economy is expanding. Due to
the higher number of funds available within the economy, consumers are more inclined to purchase
goods and services. As demand for goods and services increases but production has not yet compensated
for the shift, prices rise to restore market equilibrium.
o When the potential GDP is higher than the real GDP, the gap is instead referred to as a deflationary gap.
o The other type of output gap is the recessionary gap, which describes an economy operating below its
full-employment equilibrium.

Q 24.B
• Stem cells are unspecialized cells of the human body. They are able to differentiate into any cell of an
organism and have the ability of self-renewal. Stem cells exist both in embryos and adult cells. There are
several steps of specialization. Developmental potency is reduced with each step, which means that a
unipotent stem cell is not able to differentiate into as many types of cells as a pluripotent one.
• Totipotent stem cells are able to divide and differentiate into cells of the whole organism. Totipotency
has the highest differentiation potential and allows cells to form both embryo and extra-embryonic
structures. One example of a totipotent cell is a zygote, which is formed after a sperm fertilizes an
egg. Hence statement 3 is not correct.
• Cell differentiation: The process during which young, immature (unspecialized) cells take on individual
characteristics and reach their mature (specialized) form and function.
• Pluripotent stem cells (PSCs) form cells of all germ layers but not extraembryonic structures, such as
the placenta. Embryonic stem cells (ESCs) are an example. ESCs are derived from the inner cell mass of

9 www.visionias.in ©Vision IAS


preimplantation embryos. Another example is induced pluripotent stem cells (iPSCs) derived from the
epiblast layer of implanted embryos. Hence statement 2 is correct.
• Multipotent stem cells have a narrower spectrum of differentiation than PSCs, but they can specialize
in discrete cells of specific cell lineages. One example is a hematopoietic stem cell, which can develop
into several types of blood cells. Hence statement 1 is not correct.
• Oligopotent stem cells can differentiate into several cell types. A myeloid stem cell is an example that
can divide into white blood cells but not red blood cells.
• Unipotent stem cells are characterized by the narrowest differentiation capabilities and a special
property of dividing repeatedly. Their latter feature makes them a promising candidate for therapeutic use
in regenerative medicine. These cells are only able to form one cell type, e.g. dermatocytes.

Q 25.D
• For ensuring availability of quality products to consumers, Quality Control Orders (QCOs) are issued by
various Ministries/Departments of the Government of India in exercise of the powers conferred by
section 16 of the Bureau of Indian Standards Act, 2016 stipulating conformity of the products to Indian
Standards. Hence statement 1 is correct.
• India is looking to bring as many as 50 products such as aluminium, copper items, and household electrical
appliances under the quality norms by the second quarter of 2023-24, a move aimed at containing import
of the sub-standard goods and boost domestic industry. These orders also help in containing imports of
non-essential items. Hence statement 3 is correct.
• The items, under these orders, cannot be produced, sold/traded, imported and stocked unless they
bear the BIS mark. It said that the DPIIT is in the process of formulating QCO for various products. Hence
statement 2 is correct.

Q 26.D
• The OECD recently released a new global tax transparency framework, the Crypto-Asset Reporting
Framework (CARF), for reporting and exchange of information with respect to crypto-assets.
• This is in response to an earlier proposal of G20 that the OECD develop a framework for the automatic
exchange of information between countries on crypto-assets. The CARF will be presented to G20 Finance
Ministers and Central Bank Governors for discussion.
• The CARF will target any digital representation of value that relies on a cryptographically secure distributed
ledger or a similar technology to validate and secure transactions.
• Entities or individuals that provide services effectuating exchange transactions in crypto-assets for, or on
behalf of customers would be obliged to report under the CARF, which is expected to increase the
transparency of such transactions.
• The OECD announcement of a new transparency framework for crypto transactions is a welcome move as
it will enable timely tracking of information and ensure that regulatory and tax compliances are duly adhered
to across jurisdictions to create a level playing field.
• Hence option (d) is the correct answer.

Q 27.B
• Recent context: In April 2022, India and the U.S. signed a new pact for monitoring space objects at
the 2+2 dialogue.
• The Kessler syndrome, first put forth by NASA scientist Donald Kessler in 1978, is a scenario in
which spent rockets, satellites and other space trash that have accumulated in orbit increase the
likelihood of collision with other debris, creating a runaway chain reaction of collisions and more
debris.
• This cascade of collisions first came to NASAs attention in the 1970’s when derelict Delta rockets left in
orbit began to explode creating shrapnel clouds.
• Kessler demonstrated that once the amount of debris in a particular orbit reaches critical mass, collision
cascading begins even if no more objects are launched into the orbit.
• Once collisional cascading begins, the risk to satellites and spacecraft increases until the orbit is no longer
usable.
• The Indian Space Research Organisation (ISRO) has initiated ‘Project NETRA’ to monitor space
debris.
• The domestic surveillance system would provide first-hand information on the status of debris, which would
aid further planning on protecting space assets.
• Hence option (b) is the correct answer.

10 www.visionias.in ©Vision IAS


Q 28.D
• Corals are marine invertebrates within the class Anthozoa of the phylum Cnidaria. They typically
form compact colonies of many identical individual polyps. Coral species include the important reef
builders that inhabit tropical oceans and secrete calcium carbonate to form a hard skeleton. The Great
Barrier Reef of Australia is the largest coral reef system in the world. It has been designated a World
Heritage Site and is one of the world’s seven natural wonders. It can also be seen from outer space. Hence,
statement 1 is correct.
• All black corals have a skeleton made of protein and chitin (the same material as an insect skeleton). In
addition, black corals do not have symbiotic algae associated with them, and they do not require light
which enables them to extend into depths where light is not present. Hence, statement 2 is correct.
• Many species of corals also have growth rings similar to those of trees. Scientists can extract cores from
coral, and the coral growth rings can be used to reconstruct past climate in the tropical and subtropical
regions. Hence, statement 3 is correct.

Q 29.C
• Indian National Army (INA) was an armed force formed by Indian Nationalists in 1942, through the
patronage of the Imperial Japanese Army.
• The idea of INA was first conceived in Malaya by Mohan Singh. The outbreak of the Quit India movement
gave a fillip to INA. On 1st September, the first division of INA was formed with 16300 men.
• But due to serious differences between the INA army led by Mohan Singh and the Japanese over the role
INA was going to play, little further gain was made in its first phase.
• The second phase of the INA began when Subash Chandra Bose was brought to Singapore in 1943.
• On 21 October 1943, Bose announced the formation of the Provisional Government of Azad Hind
(Free India), with himself as the Head of State, Prime Minister, and Minister of War. Hence statement
1 is not correct and statement 2 is correct.
• The Provisional Government not only enabled Bose to negotiate with the Japanese on an equal footing but
also facilitated the mobilization of Indians in East Asia to join and support the INA. Soon after the
announcement, the Provisional Government received recognition from various countries.
• The provisional government then declared war on Britain and was recognized by the axis powers and
their satellites. In World War II, the three great Allied powers include Great Britain, the United States, and
the Soviet Union. The three principal partners in the Axis alliance were Germany, Italy, and Japan. Hence
statement 3 is correct.

Q 30.B
• WALLABY, or the Widefield ASKAP L-band Legacy All-sky Blind survey, is a radio telescope in
Western Australia that is helping astronomers build a three-dimensional map of the night sky, mapping
galaxies as far as a billion light years away. Hence pair 1 is correctly matched.
• The closest black hole to Earth has been named Gaia BH1 by astronomers using the Gemini North
telescope on the island of Hawaii, one of the twin telescopes of the International Gemini Observatory, which
is run by NSF's NOIRLab. Hence pair 2 is not correctly matched.
• Recently, a unique black hole has been discovered in a galaxy around one billion light years away from
Earth. The black hole is spewing electron-rich radio jets at another galaxy. The galaxy that hosts the black
hole is named RAD12, an elliptical galaxy that has been caught merging with a gas-poor, bigger, and
brighter elliptical galaxy. Hence pair 3 is correctly matched.

Q 31.A
• The Arctic Council is the leading intergovernmental forum promoting cooperation, coordination and
interaction among the Arctic States, Arctic Indigenous peoples and other Arctic inhabitants on
common Arctic issues, in particular on issues of sustainable development and environmental
protection in the Arctic. It was formally established in 1996 by the Ottawa declaration. Hence,
statements 1 and 2 are correct.
• Arctic Ministers decided to establish the Standing Arctic Council Secretariat at the Fram Centre in
Tromsø.
• The Ottawa Declaration defines these states as Members of the Arctic Council. The eight States have
territories within the Arctic and thus carry the role of stewards of the region. These are Canada, the Kingdom
of Denmark, Finland, Iceland, Norway, the Russian Federation, Sweden and the United States of
America. India along with the other 12 countries has an observer status in the Arctic council. Hence,
statement 3 is not correct.

11 www.visionias.in ©Vision IAS


Q 32.A
• India is the second-largest consumer of edible oil globally and its largest importer (India’s palm oil
imports account for nearly 20% of the global trade). While the government has taken several initiatives
over the years to increase the domestic production of edible oils, its shortage and high price are often in the
news. Hence, statement 3 is not correct.
• Oil Palm Development Programme (OPDP) in India was launched during 1999-2000 under the
“Technology Mission on Oil Seeds and Pulses (TMOP) with a focus on area expansion. From 2004-05
onwards the scheme is being implemented as part of the “Integrated Scheme of Oil Seeds, Pulses, Oil Palm
and Maize (ISOPOM). A special initiative was undertaken in 2011-12 under the Rashtriya Krishi Vikas
Yojana for the expansion of the area under oil palm cultivation.
• The Oil Palm grows best in areas with a mean maximum temperature of 30-32 0C and on average of at least
five hours of sunlight. Oil palm is a humid crop and requires a well-distributed rainfall of 200 cm per
annum. However, it can tolerate two to four months of dry spells. The adult palms can withstand
occasional water logging, but frequent water logging and extremely sandy and hard lateritic soils should be
avoided. Best-suited soils are moist, well-drained, deep loamy alluvial soils, rich in organic matter with
good water permeability. Hence, statement 1 is correct.
• Special features of Oil Palm
o It is a perennial crop that starts yielding from the 4th year of its planting. Hence, statement 2 is
not correct.
o Productivity per ha is more compared to other oil seeds. Under favorable conditions, it can yield 4-6
MTs of oil per ha per annum.
o The cost of cultivation is less compared to other oil seeds.
o The palm leaf can be used as fodder, as an ingredient for organic fertilizers, as firewood, etc.
o Palm oil is rich in vitamin A and Vitamin E.

Q 33.C
• Major developments in Company’s commercial privileges:
o The significant improvement in the English Company’s position was by the ‘Golden Farman’
issued to them by the Sultan of Golconda in 1632. On payment of 500 pagodas a year, they earned
the privilege of trading freely in the ports of Golconda. Hence statement 1 is not correct.
o By the Charter Act of 1793, the Company was empowered to give licences to individuals as well
as the Company’s employees to trade in India. The licenses, known as ‘privilege’ or ‘country
trade’, paved the way for shipments of opium to China. Hence statement 2 is correct
o By the Charter Act of 1813, the Company’s monopoly over trade in India ended, but the Company
retained the trade with China and the trade in tea, which eventually ended by Charter Act of
1833. Hence statement 3 is not correct.

Q 34.B
• EU members have agreed to implement a minimum tax rate of 15% on big businesses in accordance with
the global tax agreement framed by the Organisation for Economic Cooperation and Development (OECD)
last year.
• Pillar 1 of the OECD’s tax plan tries to address the question of taxing rights. Large multinational companies
have traditionally paid taxes in their home countries even though they did most of their business in foreign
countries. The OECD plan tries to give more taxing rights to the governments of countries where large
businesses conduct a substantial amount of their business. As a result, large U.S. tech companies may
have to pay more taxes to the governments of developing countries. Hence statement 1 is not correct.
• Under Pillar 2 of the global tax agreement framed by the Organisation for Economic Cooperation
and Development (OECD), countries agreed to implement a minimum tax rate of 15% on big
businesses. Under the OECD’s plan, governments will be equipped to impose additional taxes in case
companies are found to be paying taxes that are considered too low. This is to ensure that big businesses
with global operations do not benefit by domiciling themselves in tax havens in order to save on
taxes. Hence statement 2 is correct.
• Corporate tax rates across the world have been dropping over the last few decades as a result of competition
between governments to spur economic growth through greater private investments. Global corporate tax
rates have fallen from over 40% in the 1980s to under 25% in 2020. The OECD’s tax plan tries to put an
end to this “race to the bottom” which has made it harder for governments to shore up the revenues required
to fund their rising spending budgets. It is estimated that the minimum tax rate would boost global tax
revenues by $150 billion annually. Hence statement 3 is not correct.

12 www.visionias.in ©Vision IAS


Q 35.C
• Sharm El Sheikh is an Egyptian city on the southern tip of the Sinai Peninsula, located on the coastal strip
along the Red Sea. Sharm el-sheikh is a city in Egypt, the host of UNFCCC COP 27. It is a major tourist
centre in Egypt. It was in news as COP27 was held in Sharm el-Sheikh in November, 2022. Hence pair 1
is not correctly matched.
• Samarkand is a city in southeastern Uzbekistan and among the oldest continuously inhabited cities in
Central Asia. The 2022 Shanghai Cooperation Organisation summit was held in September 2022 in
Samarkand. Uzbekistan is the current chair of the organisation. It is also one of the most important sites on
the Silk Route, an ancient trade route that links China to the Mediterranean traversing Central Asia. Hence
pair 2 is correctly matched.
• Kherson is a city in southern Ukraine. It lies on the right west bank of the lower Dnieper River. Kherson
was the scene of heavy fighting during the Russian invasion of Ukraine in 2022, and it was the first
major city to fall to Russian forces. Kherson region borders Crimea and provides Moscow with a land bridge
to the Black Sea peninsula that it seized from Kyiv in 2014. Ukraine retaking swathes of Kherson region
would deprive Moscow of that land corridor. Hence pair 3 is not correctly matched.
• The Panjshir Valley s a valley in northeastern Afghanistan near the Hindu Kush mountain range. In
2021, the valley again witnessed stiff resistance against the Taliban regime. Hence pair 4 is correctly
matched.

Q 36.A
• The Nettur Petti is an artefact of historical significance. It was jewel box used by women in the royal
and aristocratic families as well as in temples to store the ornaments of the idol. This petti (box) is
named after Nettur, the village in Kerala where the craft is believed to have originated. Hence
statement 1 is correct.
• Nettur Petti requires highly skilled artistry of metal casting and the intricacy of freehand painting. It is a
true amalgamation of signature styles of handcrafting skills.
• Nettur Petti is made by the rules of ‘Tachusastram’ an ancient architecture science followed in Kerala.
Nettur Petti is also known as Malabar box or Amaadapetti in different parts of Kerala. Construction and
shape of the box is compared to a traditional Kerala house and the measurements are also followed the same
principles.
• Wood is the main component of Nettur Petti, woods mainly used are Rosewood, Aanjili, Jackwood
and Mahogany which are sourced locally. Hence statement 2 is not correct.
o Raw metal and brass sheets are the next major component used in the making of Nettur Petti.

Q 37.C
• Statement 1 is correct: The constitution in article 157 lays down only two qualifications for the
appointment of a person as a governor. These are- He should be a citizen of Indian and He should have
completed the age of 35 years.
• Statement 2 is correct: Additionally, two conventions have also developed in this regard over the years.
First, he should be an outsider, that is, he should not belong to the state where he is appointed so that he is
free from local politics. Second, while appointing the governor, the president is required to consult the
chief minister of the state concerned, so that the smooth functioning of the constitutional machinery
in the state is ensured. However, both conventions have been violated in some of the cases.

Q 38.A
• The Kisan Sabha movement gained momentum in central Andhra districts under the leadership of
the Congress Socialist Party activist N.G. Ranga. He organized a number of peasant marches in 1933-34,
and under his stewardship, at the Ellore Zamindari Ryots Conference in 1933, the demand was raised
for the abolition of zamindari.
• In 1935 Ranga and E.M.S. Namboodripad in order to spread the peasant movement to other linguistic
regions of the Madras Presidency, organized a South Indian Federation of Peasants and Agricultural
Labour and initiated the discussion for an all-India peasant body.
• E.M.S. Namboodiripad was one of the foremost leaders of the Communist movement in India and one of
the founding leaders of the Communist Party of India (Marxist).
o He left college in 1931 to join the freedom struggle and was jailed in the satyagraha movement. From
then onwards, he played an important role in the Congress movement and was one of the founders of
the Congress Socialist Party in Kerala. In 1934 he became the all-India joint secretary of the Congress
Socialist Party.

13 www.visionias.in ©Vision IAS


• Gogineni Ranga Nayukulu, better known as N. G. Ranga was an Indian freedom fighter,
parliamentarian, and Kisan (farmer) leader. He was an exponent of the peasant philosophy and was
considered the father of the Indian Peasant Movement after Swami Sahajanand Saraswati.
o Ranga joined the freedom movement inspired by Mahatma Gandhi's clarion call in 1930. He led the
ryot agitation in 1933. Three years later, he launched the Kisan Congress party. He held historic
discussions with Gandhiji on the demand for a rythu‐coolie state. He wrote a book, ‘Bapu Blesses’
regarding his discussions with Gandhi.
o Ranga was one of the founders of the Congress Socialist Party and was a member of its National
Executive and drafting Committee in 1934.
o Ranga was the first General secretary of All India Kisan Sabha formed in 1936.
• Hence option (a) is the correct answer.

Q 39.D
• Biodiversity Heritage Sites (BHS) are areas that are unique, ecologically fragile ecosystems having
rich biodiversity comprising of any one or more of the components such as; species richness, high
endemism, presence of rare, endemic and threatened species, keystone species, species of evolutionary
significance, wild ancestors of domestic/cultivated species or land races or their varieties, past pre-eminence
of biological components represented by fossil beds and having cultural or aesthetic values.
• Under Section-37 of Biological Diversity Act, 2002, the State Government in consultation with local
bodies may notify areas of biodiversity importance as Biodiversity Heritage Sites (BHS). Hence
statement 1 is not correct.
• Areas having any of the following characteristics, may qualify for inclusion as BHS.
o Areas that contain a mosaic of natural, semi-natural, and manmade habitats, which together contain a
significant diversity of life forms.
o Areas that contain significant domesticated biodiversity component and/or representative agro-
ecosystems with ongoing agricultural practices that sustain this diversity.
o Areas that are significant from a biodiversity point of view as also important cultural spaces such as
sacred groves/trees and sites, or other large community-conserved areas.
o Areas including very small ones that offer refuge or corridors for threatened and endemic fauna and
flora, such as community conserved areas or urban greens and wetlands.
o Areas that provide habitats, aquatic or terrestrial, for seasonal migrant species for feeding and breeding.
o Areas that are maintained as preservation plots by the research wing of Forest department.
o Medicinal Plant Conservation Areas.
• Recently Tamilnadu issued a notification for the first biodiversity heritage site, Arittapatti in the
state.
o This BHS has rich biological and historical significance with the presence of around 250 bird species
including 3 flagship Raptor species - Laggar Falcon, Shaheen Falcon, Bonelli's Eagle and wildlife like
Indan Pangolin, Python and Slender Loris.
• Bambarde Myristica Swamps is a biodiversity heritage site in the state of Maharashtra. Hence
statement 2 is not correct.
o Myristica swamps are freshwater swamps predominated by members of the Myristicaceae family.
These forests are characterized by trees with large protruding roots jutting out of waterlogged soil which
remains inundated throughout the year.
o They have evolved over millions of years and are comprised of old-growth trees. In India, these unique
habitats occur in the Western Ghats and a smaller distribution exists in the Andaman and Nicobar
Islands.
o The formation of these swamps is dependent on abiotic conditions like the shape of the valley between
the forested hills.
• Patalkot is a biodiversity heritage site in Madhya Pradesh.
o It has an ecosystem of the estimated age of 6 Million years and species of rare flora and fauna including
rare Bryophytes and Pteridophytes.

Q 40.B
• US dollar-denominated debt remained the largest component of India’s external debt, with a share
of 54.7 per cent at end-June 2022, followed by debt denominated in the Indian rupee (30.4 per cent), SDR
(6.3 per cent), yen (5.1 per cent), and the euro (2.8 per cent). Hence, statement 1 is correct.
• Loans remained the largest component of external debt, with a share of 31.1 per cent, followed by
currency and deposits (22.3 per cent), trade credit and advances (20.6 per cent) and debt securities (18.0 per
cent). Hence, statement 2 is correct.
14 www.visionias.in ©Vision IAS
• At end-June 2022, India’s external debt was placed at US$ 617.1 billion, recording a decrease of US$ 2.5
billion over its level at end-March 2022. At end-June 2022, long-term debt (with original maturity of above
one year) was placed at US$ 487.3 billion, recording a decrease of US$ 10.6 billion over its level at end-
March 2022 (around 78% share). Hence, statement 3 is not correct.

Q 41.D
• In May 2022, the United States launched the Indo-Pacific Economic Framework for Prosperity (IPEF) with
Australia, Brunei Darussalam, Fiji, India, Indonesia, Japan, the Republic of Korea, Malaysia, New Zealand,
Philippines, Singapore, Thailand, and Vietnam.
• This framework will advance resilience, sustainability, inclusiveness, economic growth, fairness, and
competitiveness for our economies. Through this initiative, the IPEF partners aim to contribute to
cooperation, stability, prosperity, development, and peace within the region. This framework will offer
tangible benefits that fuel economic activity and investment, promote sustainable and inclusive economic
growth, and benefit workers and consumers across the region. The 14 IPEF partners represent 40 percent
of the global GDP and 28 percent of global goods and services trade.
• The launch began discussions of future negotiations on the following pillars:
o Trade
o Supply Chains
o Clean Energy, Decarbonization, and Infrastructure
o Tax and Anti-Corruption
• India agreed to three out of four pillars, which are Supply Chains, Tax & Anti-Corruption and Clean
Energy. India stays out of 'Trade Pillar' of IPEF.
• The IPEF is designed to be flexible, meaning that IPEF partners are not required to join all four pillars.
Hence, option (d) is the correct answer

Q 42.B
• Jain Tirthankaras: In Jainism, Tirthankaras are called the Jina or the conquerors of all instincts. There are
24 Tirthankaras.
o The term, ‘Tirthankara’ is a combination of ‘Teertha and ‘Samsara’. Teertha is a pilgrimage site and
samsara is the worldly life. The one who has conquered the samsara and understood the true nature of
the self to attain Kevala Jnana is a Tirthankara.
o Definition of a Tirthankar: A Tirthankar is referred to as ‘teaching god’ or ‘Ford Maker’ in Jainism.
o The names of 24 Tirthankaras are inspired by the dreams their respective mothers had before their birth
or related circumstances surrounding their births.
o Kalpasutra is a religious text of Jains that mentions the life histories of 24 Tirthankaras. (It is
apparently compiled by Digambara sect Jain Muni Bhadrabahu 150 years after Mahavir’s Nirvana.)
✓ Kalpasutra mentions the first Tirthankara to be Rishabhnath.
Jain Tirthankara Symbol
Adinatha/Rishabnatha Ox/Bull
Parshvanatha Snake/Serpent
Mahavira Lion
Neminath Conch
• Hence only pairs 2 and 3 are correctly matched.
• Facts about Most Prominent Tirthankaras
o Rishabhnath: He is said to exist before Indus Valley Civilization. It is mentioned that in Bhagavata
Purana, he is referred to as Lord Vishnu. Vedas also mention the name of Rishabhnath. He had many
sons including – Bharat and Bahubali (Note: The Gomateshwara Statue is dedicated to Bahubali, and
is the tallest statue in the world. It is located in Shravanabelagola in Karnataka.). It is also believed that
the name of the script ‘Brahmi’ is inspired by his daughter’s name.
o Parshvanatha: Parsvanath was the 23rd Tirthankara. It is believed that he existed two centuries before
Vardhamana Mahavira. He was apparently born in Banaras (Uttar Pradesh) around 817 BCE. He is said
to have propounded the Jain religion which was later revived by Mahavira. He attained Kaivalya on
Mount Sammeta (Parasnath) in Jharkhand. According to the Svetambaras Sect (White-Clad Sect of
Jainism); Parshvanatha founded four-fold restraints:
✓ Ahimsa
✓ Satya
✓ Asteya
✓ Aparigraha (The fifth one, ‘Brahmacharya’ was added by Mahavira.)
15 www.visionias.in ©Vision IAS
✓ Navagraha Jain Temple in Karnataka houses the tallest statue of Parsvanath.
o Mahavira: He was the 24th Tirthankara of the Jain religion. He was the son of Siddhartha and Trishla.
He was born in Bihar. He was a contemporary of Gautam Buddha. At the age of 30, he left his worldly
possessions and sought ascetic life towards Kevala Gnan. He attained Kaivalya under a Sal tree. He
attained Nirvana in Pavapuri, Bihar.

Q 43.A
• The presiding officer of the Rajya Sabha is known as the Chairman.
• The vice-president of India is the ex-officio Chairman of the Rajya Sabha.
• During any period when the Vice-President acts as President or discharges the functions of the President,
he does not perform the duties of the office of the Chairman of Rajya Sabha.
• The Chairman of the Rajya Sabha can be removed from his office only if he is removed from the office of
the Vice-President.
• As a presiding officer, the powers and functions of the Chairman in the Rajya Sabha are similar to those of
the Speaker in the Lok Sabha. However, the Speaker presides over a joint sitting of two Houses of
Parliament and the Chairman of Rajya Sabha can not preside over a joint sitting of the two Houses of
Parliament under any circumstances. Hence statement 3 is correct.
• Unlike the Speaker (who is a member of the House), the Chairman is not a member of the House. The
Chairman can not vote in the first instance. However, he can cast a vote in the case of an equality of
votes. Hence statement 2 is not correct.
• The Vice-President cannot preside over a sitting of the Rajya Sabha as its Chairman when a
resolution for his removal is under consideration. However, he can be present and speak in the House
and can take part in its proceedings, without voting, even in the case of an equality of votes (while the
Speaker can vote in the first instance when a resolution for his removal is under consideration of the Lok
Sabha). Hence statement 1 is correct.
• The salaries and allowances of the Chairman are fixed by the Parliament. They are charged on the
Consolidated Fund of India and thus are not subject to the annual vote of Parliament.
• During any period when the Vice-President acts as President or discharges the functions of the President,
he is not entitled to any salary or allowance payable to the Chairman of the Rajya Sabha. But he is paid the
salary and allowance of the President during such a time.

Q 44.A
• Composition of Rajya Sabha
o The Rajya Sabha has a maximum strength of 250 members, of which 238 are representatives of states
and union territories (elected indirectly) and 12 are nominated by the president for their excellence
in the field of literature, science, art, and social service.
o The Rajya Sabha currently has 245 members. Hence, statement 1 is not correct.
o The states are represented by 229 members, the union territories are represented by 4 members,
and the president appoints 12 members.
o The vice-President of India is also the ex-officio chairman of the Rajya Sabha. He or she presides over
Rajya Sabha meetings.
o In his absence, the meeting of the House is presided over by the Deputy Chairman, who is elected by
its members from among themselves.
o The Deputy Chairman can be removed by a majority of all Rajya Sabha members at the time.
• Representation of States
o The Rajya Sabha's representatives are elected by the members of state legislative assemblies.
o The election is held using a single transferable vote in accordance with the proportional representation
system.
o The Rajya Sabha seats are allocated to the states based on population. As a result, the number of
representatives varies by state.
• Representation of Union Territories
o The representatives of each union territory in the Rajya Sabha are indirectly elected by members of a
special electoral college.
o This election is also held in accordance with the proportional representation system through the use of
a single transferable vote.
o Only three of the eight union territories (Delhi, Puducherry, Jammu, and Kashmir) have representation
in Rajya Sabha. Hence, statement 2 is not correct.
o In total eight members are elected from the Union territories (3 from Delhi, 1 from Puducherry,
and 4 from Jammu & Kashmir).
16 www.visionias.in ©Vision IAS
o Other Union territories are not represented in Rajya Sabha.
o Maximum representation is from Jammu and Kashmir. Hence, statement 3 is correct.
o The other five union territories' population is too small to have a representative in the Rajya Sabha.
Q 45.D
• The Coriolis effect describes the pattern of deflection taken by objects not firmly connected to the ground
as they travel long distances around Earth. The Coriolis effect is responsible for many large-scale weather
patterns.
• The key to the Coriolis effect lies in Earth’s rotation. Specifically, Earth rotates faster at the Equator than
it does at the poles. Earth is wider at the Equator, so to make a rotation in one 24-hour period, equatorial
regions race nearly 1,600 kilometers (1,000 miles) per hour. Near the poles, Earth rotates at a sluggish
0.00008 kilometers (0.00005 miles) per hour.
• Military snipers sometimes have to consider the Coriolis effect. Although the trajectory of bullets is too
short to be greatly impacted by Earth’s rotation, sniper targeting is so precise that a deflection of several
centimeters could injure innocent people or damage civilian infrastructure.
• The weather impacting fast-moving objects, such as airplanes and rockets, is influenced by the Coriolis
effect. The directions of prevailing winds are largely determined by the Coriolis effect, and pilots must
take that into account when charting flight paths over long distances.
• The development of weather patterns, such as cyclones and trade winds, are examples of the impact
of the Coriolis effect. Cyclones are low-pressure systems that suck air into their center, or “eye.” In the
Northern Hemisphere, fluids from high-pressure systems pass low-pressure systems to their right. As air
masses are pulled into cyclones from all directions, they are deflected, and the storm system—a hurricane—
seems to rotate counter-clockwise. In the Southern Hemisphere, currents are deflected to the left. As a result,
storm systems seem to rotate clockwise. Outside storm systems, the impact of the Coriolis effect helps
define regular wind patterns around the globe.
• Hence, option (d) is the correct answer.

Q 46.A
• Recently a rare flock of Wilson’s Storm Petrels were found off Alappuzha coast, Kerala.
o Wilson’s Storm Petrel is a small seabird with a swallow-like shape: long, fairly broad wings and a
short tail. Wilson's Storm-Petrels have long legs and their feet project beyond the tail tip. The head is
small and round, with a short bill with tubelike nostrils on top
o Wilson's Storm-Petrel is one of the widest-ranging of all seabirds. It breeds as far south as Antarctica,
but regularly travels to all of the world's oceans except the North Pacific and Arctic.
o Its IUCN status is least concern.
• Recently, a great knot from Russia, belonging to the endangered Calidris tenuirostris (Horsfield,
1821), has found its way to Kerala’s coast, flying over 9,000 km for a winter sojourn.
o The migratory bird that traversed the Central Asian Flyway (CAF) is only one of the two — the other
has been sighted at Jamnagar in Gujarat — great knots to be re-sighted in India among the nearly
thousand ones tagged with MOSKVA rings in the Kamchatka peninsula in eastern Russia.
o The great knot (Calidris tenuirostris) belongs to the family of sandpipers and knots, the Scolopacidae.
The great knot is distributed in far northeast Russia, coastal Australia, southeast Asia, India,
Bangladesh, Pakistan and eastern Arabian Peninsula. These knot species are listed as "Endangered" by
IUCN.
• Recently, Indian skimmer was seen in huge flocks during winter in Coringa wildlife sanctuary
(Kakinada)
• Indian Skimmer is recently uplisted as Endangered on the IUCN Red list. It has gone a substantial decline
throughout South East Asia. It was formerly widely distributed across the Indian Subcontinent but it is
presently confined only to India, Pakistan, Bangladesh, Nepal and Myanmar. It is considered as extinct in
Lao PDR, Cambodia and Vietnam.
o About 20% of the total population of fewer than 2,500 birds nest along river Chambal. Villagers here
call it pancheraa, that which tears water.
• Hence option (a) is the correct answer.

Q 47.D
• A soft landing, in economics, is a cyclical slowdown in economic growth that avoids recession. A soft
landing is the goal of a central bank when it seeks to raise interest rates just enough to stop an economy
from overheating and experiencing high inflation, without causing a severe downturn. Soft landing may
also refer to a gradual, relatively painless slowdown in a particular industry or economic sector.
• Hence, option (d) is the correct answer.
17 www.visionias.in ©Vision IAS
Q 48.D
• Aerosols are minute particles suspended in the atmosphere. They are short-lived, unlike greenhouse
gases that persist and accumulate in the atmosphere for longer period. Aerosols are non-uniform in shape
and size so equivalent aerodynamic diameter determined by comparing them with perfect spheres having
same settling velocity. Aerosols come from volcanoes, dust storms, fires, vegetation, sea spray, burning of
fossil fuels and land use. Hence statement 1 is correct.
• Aerosols interact both directly and indirectly with the Earth's radiation budget and climate.
o Different aerosols scatter or absorb sunlight to varying degrees, depending on their physical properties.
Although most aerosols reflect sunlight, some also absorb it.
o As an indirect effect, aerosols in the lower atmosphere can modify the size of cloud particles, changing
how the clouds reflect and absorb sunlight, thereby affecting the Earth's energy budget. Hence
statement 3 is correct.
• A good monsoon is produced by the difference in temperature between land and sea. But, the dust clouds
(formed by aerosols) shield the earth from the sun’s rays, depressing land and sea temperatures and reducing
the variation between the two. Because of this, the Indian monsoon is getting weakened by aerosol
accumulation.
• Contrary to convectional rainfall, in orographic rainfall when moist air gets obstructed, the air raises
and gets adiabatically cooled leading to condensation, cloud formation and rain. With the presence of
high Aerosols they serve as cloud condensation Nuclei resulting in high incidence of cloud formation and
rainfall. Hence statement 2 is correct.
o As Indo-Gangetic Plain, one of the world’s most polluted areas, is upwind of the Himalayan foothills
and the presence of Aerosols in high concentration are leading to high incidence of rainfall in foothills
of Himalayas.
Q 49.D
• The Hausas, settled cultivators:
o The Hausa are a tribe of settled cultivators who inhabit the savanna-lands of the Bauchi Plateau of
northern Nigeria. They number almost six million and have been organised in settled agricultural
communities for hundreds of years.
o They are more advanced in their civilisation and ways of life than many of the other African tribes. For
instance, they live in towns or villages and do not practise shifting cultivation as many tribes do. Instead
they clear a piece of land and use it for several years, growing a wide range of crops like maize, millet,
groundnuts etc. They also cultivate non food crops like cotton and tobacco.
o Besides cultivation, the Hausa also make use of domesticated animals, but they are only subsidiary to
crop cultivation.
o Hence pair 1 is correctly matched.
• The Eskimos:
o Greenland, northern Canada and Alaska are their lands. They used to live as hunters, fishers and food-
gatherers but in recent years more and more of them are settling in permanent huts. During winter they
live in compact igloos and in summer when they move out to hunt they pitch portable tents of skins by
the side of streams
o The Polar Eskimos, living around Thule in north-west Greenland still lead an uncertain life, not very
much different from their forefathers.
o Hence pair 2 is correctly matched.
• Zulus:
o The Zulu are the largest South African ethnic group, with an estimated 10–11 million people living
mainly in the province of KwaZulu-Nata and small numbers also live in Zimbabwe, Zambia and
Mozambique.
o The language of Zulus known as isiZulu, is a Bantu language; more specifically, part of the Nguni
subgroup. They remain today the most numerous ethnic group in South Africa, and now have equal
rights along with all other citizens. They are considered warriors.
o Hence pair 3 is correctly matched.

Q 50.D
• The government on 24/04/2018 on the occasion of National Panchayati Raj Day launched the
restructured Centrally Sponsored Scheme (CSS) of Rashtriya Gram Swaraj Abhiyan (RGSA).
Hence, statement 2 is not correct.
• Its primary aim is strengthening Panchayati Raj Institutions (PRIs) for achieving Sustainable
Development Goals (SDGs) with the main thrust on convergence with Mission Antyodaya and
emphasis on strengthening PRIs in the 117 Aspirational districts. Hence, statement 1 is correct.
18 www.visionias.in ©Vision IAS
• The scheme has been approved with a total budget outlay of Rs. 7255.50 crores out of which the State share
will be Rs. 2755.50 crores and the Central share will be Rs. 4500.00 crores. The scheme extends to all States
and UTs including non-Part IX areas where Panchayats do not exist.
• The scheme consists of Central and State Components.
• The Central component comprises
o National level activities viz. National Plan for Technical Assistance (NPTA) including NPMU,
collaboration with academic institutions/ institutions of excellence including NIRD&PR, Hyderabad
for various activities of CB&T for PRIs
o Mission Mode Project (MMP) on e-Panchayat and
o Incentivization of Panchayats.
• The State component relates to activities to be undertaken by State Governments for CB&T and other
activities for strengthening of Panchayats viz Capacity Building &Training, training infrastructure and HR
support for training, Strengthening of Gram Sabhas in PESA areas, Distance Learning Facility via
SATCOM, Support for Innovations, Technical support to PRIs, Financial Data and Analysis Cell, Panchayat
Buildings, e-enablement of Panchayats, Project-based funding for economic development and income
enhancement, IEC and PMU.
• The sharing pattern for the State component is in the ratio of 60:40 except for NE and Hilly States where
Central and State sharing is in the ratio of 90:10. For all UTs, the Central share is 100%.
• The scheme comes under the Ministry of Panchayati Raj. Hence, statement 3 is not correct.

Q 51.A
• The European Union (EU) has recently given final approval to online safety-focused legislation, which is
an overhaul of the region’s social media and e-commerce rules. Called the Digital Services Act (DSA), the
law tightly regulates the way intermediaries, especially large platforms such as Google, Meta, Twitter, and
YouTube, function in terms of moderating user content.
• The European Commission has passed two legislations to upgrade rules governing digital services in
the EU: the Digital Services Act (DSA) and the Digital Markets Act (DMA).
• As defined by the EU Commission, the DSA is “a set of common rules on intermediaries’ obligations and
accountability across the single market”, and ensures higher protection to all EU users, irrespective of their
country.
• The Digital Markets Act (DMA), which came into force on Tuesday in November 2022, aims to put an end
to unfair practices by companies that act as so-called digital “gatekeepers”. The EU hopes it will help start-
ups innovate and compete with Big Tech and will give consumers more choices. Hence, option (a) is the
correct answer.

Q 52.C
• Union Minister for Rural Development and Panchayati Raj quoting figures from the National Crime
Records Bureau's (NCRB) 'Crime in India 2021' report expressed serious concern that India registered
31,677 cases of rape in 2021 - an average 86 daily - while nearly 49 cases of crime against women were
lodged every single hour. The number of rape cases in 2020 was 28,046, while it was 32,033 in 2019.
• Minister was speaking after the Launch of “A Community-led National Campaign Against Gender-
Based Discrimination” of Deendayal Antyodaya Yojana-National Rural Livelihoods Mission in New
Delhi. Hence, option (c) is the correct answer.
• The month-long campaign titled, “Nai Chetna-Pahal Badlav Ki” with the theme of ‘Elimination of
Gender-Based Violence’ will be conducted as a 'Jan Andolan' (people's movement) in all the
States/UTs of the country from 25th November to 23rd of December, 2022.
• This will be an annual campaign focussing on specific gender issues each year. The focus area of the
campaign this year is Gender-Based Violence. This campaign will be implemented by all states in
collaboration with CSO partners, and actively executed by all levels including the State, District, and Block
engaging the Community Institutions along with the extended community.
• The Campaign will also bring together all line departments and stakeholders to create a concerted effort in
acknowledging, identifying and addressing issues of violence. Over the years this campaign will incorporate
an intersectional approach deepening the understanding of gender and generating relevance and ownership
in all departments and verticals with a multisectoral approach. As part of the campaign, there will be
knowledge workshops, leadership training, and seminars on sexual violence and redressal mechanisms.

19 www.visionias.in ©Vision IAS


Q 53.A
• During the medieval period, in astronomy, a number of commentaries dealing with the already established
astronomical notions appeared.
• Firoz Shah Tughaq established observation posts at Delhi. Firoz Shah Bahmani under Hakim Hussain Gilani
and Syed Muhammad Kazimi set up an observatory in Daulatabad. Both lunar and solar calendars were in
use.
• Mehendra Suri, a court astronomer of Firoz Shah Tughlaq developed an astronomical instrument
called Yantraja.
• Parameshvara and Mahabhaskariya were famous families of astronomers and almanac-makers.
• Nilakantha Somasutvan produced a commentary on Aryabhatta.
• Kamalakar studied the Islamic ideas on astronomy. He was an authority on Islamic knowledge as well.
• Hence option (a) is the correct answer.

Q 54.C
• Recent context: FAO report finds most black soils moderately to severely eroded due to land-use change,
and unsustainable practices.
• As per the report, black soils are mineral soils that have a black surface horizon, enriched with organic
carbon that is at least 25 cm deep.
o Black color is a result of the accumulation of organic matter originating from numerous dying roots of
gramineous vegetation in a process known as melanization. Hence statement 3 is correct.
• Black soils are extremely fertile and can produce high agricultural yields due to their elevated
moisture storage capacity. Hence statement 1 is not correct.
o Chernozem is also known as a type of black soil that is black in color and is rich in nutrients. The
chernozem soil is rich in humus ranging between 4% to 16% along with a higher concentration of
nutrients like phosphoric acids, phosphorus, and ammonia. This makes the chernozem soil very fertile
and hence one of the most useful soils for agriculture and results in high agricultural yields.
• Black soils constitute 5.6 percent of global soils and contain 8.2 percent of the world’s soil organic carbon
(SOC) stocks: Approximately 56 billion tonnes of carbon. Hence statement 2 is correct.
o However, despite representing a small portion of the world’s soils, black soils were key for food security
and the global economy. Globally in 2010, 66 percent of sunflower seeds, 51 percent of small millet,
42 percent of sugar beet, 30 percent of wheat, and 26 percent of potatoes were harvested from black
soils.
• Black soils were home to 2.86 percent of the global population and had 17.36 percent of cropland, 8.05
percent of global SOC stock, and 30.06 percent of SOC stock of global cropland, according to the report.
o This signifies their importance for climate change mitigation and adaptation. However, black soils are
quickly losing their SOC stocks. They have lost 20 to 50 percent of their original SOC stock, with the
carbon being released into the atmosphere mostly as carbon dioxide, exacerbating global warming, the
report pointed out while quoting previous studies.
o Black soils have the potential to provide 10
percent of the total SOC sequestration
globally if they receive proper attention.
• Chemical Composition of Black Soils:
o 10 per cent of alumina,
o 9-10 percent of iron oxide,
o 6-8 percent of lime and magnesium
carbonates,
o Potash is variable (less than 0.5 percent) and
o phosphates, nitrogen, and humus are
low.
• Distribution of Black Soils in India:
o Spread over 46 lakh sq km (16.6 percent
of the total area) across Maharashtra, Madhya Pradesh, parts of Karnataka, Telangana, Andhra
Pradesh, Gujarat, and Tamil Nadu.
Q 55.C
• About Influenza viruses
o Influenza viruses belong to the family Orthomyxoviridae and have a single-stranded segmented
RNA genome. Hence statement 1 is not correct.
o Influenza virus is transmitted primarily by droplets or respiratory secretions of infected persons.
Hence statement 2 is correct.
20 www.visionias.in ©Vision IAS
• There are four types of influenza viruses: types A, B, C and D:
o Influenza A viruses infect humans and many different animals. The emergence of a new and very
different influenza A virus with the ability to infect people and have sustained human-to-human
transmission can cause an influenza pandemic.
o Influenza B viruses circulate among humans and cause seasonal epidemics. Recent data showed seals
also can be infected.
o Influenza C viruses can infect both humans and pigs but infections are generally mild and are rarely
reported.
o Influenza D viruses primarily affect cattle and are not known to infect or cause illness in people.
Hence statement 3 is correct.
Q 56.D
• Tax buoyancy explains this relationship between the changes in the government’s tax revenue growth and
the changes in GDP. It refers to the responsiveness of tax revenue growth to changes in GDP. When a tax
is buoyant, its revenue increases without increasing the tax rate. The numerical estimate of tax buoyancy is
very useful to understand the revenue performance of the economy. Hence statement 2 is not correct.
• A similar looking concept is tax elasticity. Tax elasticity refers to changes in tax revenue in response to
changes in tax rates. Tax elasticity is the degree to which the increase in the tax rate causes a change in the
tax base. Hence statement 1 is not correct.
• Tax elasticity is thought of as being more prevalent at higher rates. This means that when tax rates are first
increased, the tax base isn’t negatively affected. However, once the rates go higher, the negative impact on
the tax rate becomes more apparent

Q 57.D
• Lord Birkenhead, the Conservative Secretary of State responsible for the appointment of the Simon
Commission, had constantly harped on the inability of Indians to formulate a concrete scheme of
constitutional reforms which had the support of wide sections of Indian political opinion.
o This challenge, too, was taken up and meetings of the All-Parties Conference were held in
February, May and August 1928 to finalize a scheme which popularly came to be known as the
Nehru Report after Motilal Nehru, its principal author.
• Recommendations:
o This report defined Dominion Status as the form of government desired by India.
o It rejected the principle of separate communal electorates on which previous constitutional reforms had
been based.
o Seats would be reserved for Muslims at the Centre and in provinces in which they were in a minority,
but not in those where they had a numerical majority.
o The Report also recommended universal adult suffrage, equal rights for women, freedom to form
unions, and dissociation of the state from religion in any form. Hence options 1, 2 and 3 are
correct.
o Linguistic provinces.
o Responsible government at the Centre and in provinces—
✓ The Indian Parliament at the Centre to consist of a 500-member House of Representatives elected
on the basis of adult suffrage, a 200-member Senate to be elected by provincial councils; the House
of Representatives to have a tenure of 5 years and the Senate, one of 7 years; the central government
to be headed by a governor-general, appointed by the British government but paid out of Indian
revenues, who would act on the advice of the central executive council responsible to the
Parliament.
✓ Provincial councils to have a 5-year tenure, headed by a governor acting on the advice of the
provincial executive council.
o Full protection to the cultural and religious interests of Muslims.

Q 58.D
• Article 280 of the Constitution of India: The President shall, within two years from the commencement
of this Constitution and thereafter at the expiration of every fifth year or at such earlier time as the
President considers necessary, by order constitute a Finance Commission which shall consist of a
Chairman and four other members to be appointed by the President. Hence statements 1 and 2 are
not correct.
• Parliament may by law determine the qualifications which shall be requisite for appointment as
members of the Commission and the manner in which they shall be selected. Hence statement 3 is
correct.
21 www.visionias.in ©Vision IAS
• It shall be the duty of the Commission to make recommendations to the President as to—
o the distribution between the Union and the States of the net proceeds of taxes which are to be, or maybe,
divided between them under this Chapter and the allocation between the States of the respective shares
of such proceeds;
o the principles which should govern the grants-in-aid of the revenues of the States out of the
Consolidated Fund of India;
• The recommendations made by the Finance Commission are only of advisory nature and hence, not
binding on the government. It is up to the Union government to implement its recommendations on
granting money to the states.
• The Commission can determine its procedure and in the performance of its functions can have all the powers
of a civil court under the Code of Civil Procedure, 1908.

Q 59.B
• The Intergovernmental Science-Policy Platform on Biodiversity and Ecosystem Services (IPBES) is an
independent intergovernmental body established by States to strengthen the science-policy interface for
biodiversity and ecosystem services for the conservation and sustainable use of biodiversity, long-term
human well-being and sustainable development.
• It was established in Panama City, on 21 April 2012 by 94 Governments. It is not a United Nations
body. However, at the request of the IPBES Plenary and with the authorization of the UNEP
Governing Council in 2013, the United Nations Environment Programme (UNEP) provides
secretariat services to IPBES. Hence statement 1 is correct.
• The governing body of IPBES is made up of the representatives of IPBES member states and usually meets
once per year.
• IPBES currently has close to 140 member States. A large number of NGOs, organizations, conventions and
civil society groupings also participate in the formal IPBES process as observers.
• The work of IPBES can be broadly grouped into four complementary areas:
o Assessments: On specific themes (e.g. “Pollinators, Pollination and Food Production”); methodological
issues (e.g. “Scenarios and Modelling); and at both the regional and global levels (e.g. “Global
Assessment of Biodiversity and Ecosystem Services”).
o Policy Support: Identifying policy-relevant tools and methodologies, facilitating their use, and
catalyzing their further development.
o Building Capacity & Knowledge: Identifying and meeting the priority capacity, knowledge and data
needs of our member States, experts and stakeholders.
o Communications & Outreach: Ensuring the widest reach and impact of our work.
• IPBES is funded through generous voluntary contributions from its member States to the IPBES Trust Fund.
• India is a member of this organization. Hence statement 3 is correct.
• The Economics of Ecosystems and Biodiversity (TEEB) was a study led by Pavan Sukhdev from 2007
to 2011. It is an international initiative to draw attention to the global economic benefits of biodiversity. Its
objective is to highlight the growing cost of biodiversity loss and ecosystem degradation and to draw
together expertise from the fields of science, economics and policy to enable practical actions.
o In 2011, UNEP hosted the TEEB office at the United Nations Environment Programme in Geneva,
Switzerland and set up an office to manage day-to-day operations and communications activities.
o Hence statement 2 is not correct.

Q 60.D
• Recent context: 5G services in India were formally launched by Prime Minister Narendra Modi on
1st October 2022, at the 6th edition of India Mobile Congress (IMC).
• 5G or fifth generation mainly works in 3 bands, namely low, mid and high-frequency spectrum.
• While the low band spectrum has shown great promise in terms of coverage and speed of Internet and data
exchange, the maximum speed is limited to 100 Mbps.
• The mid-band spectrum, on the other hand, offers higher speeds compared to the low band, but has
limitations in terms of coverage area and signal penetration.
• The high-band spectrum offers the highest speed of all three bands, but has extremely limited coverage and
signal penetration strength. Hence statement 1 is correct.
• Since several emerging applications in factory automation, gaming, and remote healthcare have more
stringent latency requirements, 5G places a strong emphasis on low latency. Self-driving cars are an
illustrative example. Low delays between transmission and reception of messages are extremely critical
when these cars have to cooperate with each other to avoid accidents.

22 www.visionias.in ©Vision IAS


• The amount of data is also expected to grow exponentially as 5G is deployed. To prevent a similar rise in
energy usage, 5G places a lot of importance on energy efficiency. Hence statement 2 is correct.
• As 5G services evolve to occupy higher frequencies, the design of the transmitting and receiving equipment
becomes more complex. Signal attenuation also increases. So, the coverage area of each cell tower will
decrease, which will require the towers to be more closely spaced. Hence statement 3 is correct.

Q 61.D
• Recent context: In some places in Kerala, the widespread growth of forked fanwort has painted the water
bodies pink.
• It is a submerged perennial aquatic plant that grows in stagnant to slow-flowing freshwater.
o It is an invasive species that belongs to Central and South America. It requires a considerable
amount of oxygen to grow, & that could badly affect freshwater. Hence option (d) is the correct
answer.
• It is a common aquarium plant and its widespread trade in the industry has led to its proliferation to areas
outside of its native range. The main pathways of dispersal are humans by intentional aquarium plantings
and via inappropriate disposal. It has beautiful flowers and dissected leaves, and is commonly used in
aquaria.
• The plant has a high natural dispersal potential due to its ability to readily fragment and spread both actively
and passively. Its main habitats include freshwater bodies such as rivers, reservoirs, ponds, irrigation canals
and other shallow wetlands.

Q 62.B
• Abdur Razzak, like Nicolo di Conti, visited the city during the reign of Deva Raya II, but about twenty
years later than Conti. He was entrusted with an embassy from Persia, and set out on his mission on
January 13, A.D. 1442.
o Abdur Razzaq was a reluctant traveller, who left Herat only at the order of his monarch, and who swore
in his travelogue that he would never make a voyage again.
o In 1441, Shah Rukh of Persia sent Kamal-ud-din Abdur Razzaq as an emissary to Vijayanagar.
Abdur Razzaq wrote his travels in the Matla-us-Sadain wa Majma-ul-Bahrain, or The Rise of Two
Auspicious Constellations and the Confluence of Two Oceans, which was translated into French—
and then, in 1855, the translation was translated into English by R.H. Major. Hence option (b) is the
correct answer.
o His views about the Vijaynagar Empire are given below:
✓ Abdur Razzaq who had travelled widely in and outside India, and was an ambassador at the court
of Deva Raya II (1423-46), says: “This latter prince has in his dominions three hundred ports, each
of which is equal to Calicut, and his territories comprise a space of three months journey.”
✓ “The country is for the most part well cultivated, very fertile. The troops amount in number to
eleven lakhs.”
✓ He mentioned about the fortification of the city. He wrote that ''the city of Bijanagar is such that
eye has not seen nor ear heard of any place resembling it upon the whole earth. It is so built that it
has seven fortified walls, one within the other.”
• The wealth of the Vijayanagar city and its kings are attested by another Persian historian, Ferishta, the
author of Tarikh-i Ferishta, “The princes of the Bahmani maintained their superiority by valour only; for
in power, wealth and extent of the country, the Rayas of Bijarnagar (Vijayanagar) greatly exceeded
them.” Ferishta had joined the service of King Ibrahim Adil II of Bijapur in 1589.
• Al-Biruni was born in 973 AD, in Khwarizm in present-day Uzbekistan. In 1017, when Sultan Mahmud
invaded Khwarizm, he took several scholars and poets back to his capital, Ghazni; Al-Biruni was one of
them. He arrived in Ghazni as a hostage, but gradually developed a liking for the city, where he spent the
rest of his life until his death at the age of 70.
o Al-Biruni spent years in the company of Brahmana priests and scholars, learning Sanskrit, and studying
religious and philosophical texts. While his itinerary is not clear, it is likely that he travelled widely in
the Punjab and parts of northern India.
o Al-Biruni’s Kitab-ul-Hind, written in Arabic, is a voluminous text, divided into 80 chapters on
subjects such as religion and philosophy, festivals, astronomy, alchemy, manners and customs, social
life, weights and measures, iconography, laws and metrology. Generally (though not always), Al-Biruni
adopted a distinctive structure in each chapter, beginning with a question, following this up with a
description based on Sanskritic traditions, and concluding with a comparison with other cultures.
o He was familiar with translations and adaptations of Sanskrit, Pali and Prakrit texts into Arabic – these
ranged from fables to works on astronomy and medicine.
23 www.visionias.in ©Vision IAS
• By the time Ibn Battuta arrived in Delhi in the fourteenth century, the subcontinent was part of a global
network of communication that stretched from China in the east to north-west Africa and Europe in the
west.
o Travelling overland through Central Asia, Ibn Battuta reached Sind in 1333. He had heard about
Muhammad bin Tughlaq, the Sultan of Delhi, and lured by his reputation as a generous patron of arts
and letters, set off for Delhi, passing through Multan and Uch. The Sultan was impressed by his
scholarship, and appointed him the qazi or judge of Delhi.

Q 63.A
• The Gupta period in ancient India is referred to as the “Golden Age” because of the numerous achievements
in the field of arts, literature, science, and technology. It also brought about the political unification of the
subcontinent.
• Samudragupta (335-375 AD) of the Gupta dynasty is known as the Napoleon of India. Historian A V
Smith called him so because of his great military conquests known from the 'Prayag Prashati' (Allahabad
Inscription) written by his courtier and poet Harisena, who also describes him as the hero of a hundred
battles. He has been hailed for the unification of the greater part of the Indian subcontinent.
• He was a devotee of Vishnu though a follower of the Brahmanical religion. He granted permission to the
Buddhist king Meghavarman (King of Ceylon) to build monasteries to Bodh Gaya.
• It is also mentioned that Samudragupta liked playing the lute and loved listening to poems. He was titled
‘Kaviraj’ for his love for poems.
• Hence option (a) is the correct answer.

Q 64.C
• Electronic Voting Machines: In 1989, a provision was made to facilitate the use of Electronic Voting
Machines (EVMs) in elections. The EVMs were used for the first time in 1998 on an experimental basis
in selected constituencies in the elections to the Assemblies of Rajasthan, Madhya Pradesh, and Delhi. The
EVMs were used for the first time in the general elections (entire state) to the Assembly of Goa in 1999.
• Voting through Postal Ballot: In 1999, a provision was made for voting by certain classes of persons
through postal ballot.
• Free Supply of Electoral Rolls: According to a 2003 provision, the Government should supply, free
of cost, copies of the electoral rolls and other prescribed material to the candidates of recognized
political parties for the Lok Sabha and Assembly elections.
• Restrictions Imposed on Exit Polls: According to a 2009 provision, conducting exit polls and publishing
results of existing polls would be prohibited during the election to Lok Sabha and State Legislative
Assemblies.
• Hence option (c) is the correct answer.

Q 65.C
• The Suhrawardi Silsilah
o This silsilah was founded by Sheikh Shihabuddin Suhrawardi. It was established in India by Sheikh
Bahauddin Zakariya (1182-1262). He set up a leading khanqah in Multan, which was visited by rulers,
high government officials, and rich merchants. Sheikh Bahauddin Zakariya openly took Iltutmish’s side
in his struggle against Qabacha and received from him the title Shaikhul Islam (Leader of
Islam). Hence, statement 1 is correct.
o It must be noted that, unlike the Chishti saints, the Suhrawardis maintained close contact with the
state. They accepted gifts, jagirs, and even government posts in the ecclesiastical department. Hence,
statement 2 is correct.
o The Suhrawardi silsilah was firmly established in Punjab and Sind. Besides these two silsilahs there
were others such as the Firdawsi Silsilah, Shattari Silsilah, Qadiri Silsilah, Naqshbandi Silsilah.

Q 66.B
• According to the IUCN (2004), the total number of plant and animal species described so far is slightly
more than 1.5 million, but we have no clear idea of how many species are yet to be discovered and described.
• More than 70 percent of all the species recorded are animals, while plants (including algae, fungi,
bryophytes, gymnosperms and angiosperms) comprise no more than 22 percent of the total. Hence
statement 1 is not correct.
• Among animals, insects are the most species-rich taxonomic group, making up more than 70 percent
of the total. Hence statement 2 is correct.
• That means, out of every 10 animals on this planet, 7 are insects.
24 www.visionias.in ©Vision IAS
• The number of fungi species in the world is more than the combined total of the species of fishes,
amphibians, reptiles and mammals. Hence statement 3 is correct.
o Fungi include any of about 144,000 known species of organisms of the kingdom Fungi, which
includes the yeasts, rusts, smuts, mildews, molds, and mushrooms.

Q 67.C
• The earliest of the dynastic coins relate to those of the Indo-Greeks, the Saka-Pahlavas and the
Kushans. These coins are generally placed between the 2nd century BC and the 2nd century AD. Hellenistic
traditions characterize the silver coins of the Indo-Greeks, with Greek gods and goddesses figuring
prominently, apart from the portraits of the issuers. These coins with their Greek legends are historically
significant, as the history of the Indo-Greeks has been reconstructed almost entirely on their evidence.
• The Saka coinage of the Western Kshatrapas is perhaps the earliest dated coins, the dates being given
in the Saka era which commences in AD 78. The Saka era represents the official calendar of the Indian
Republic. Hence, statement 1 is correct.
o The term Western Kshatraps alludes to the set of rulers who ruled Western India between the 1st and
4th century AD. The legends on the coins were generally in Greek and Brahmi. Kharoshti too was used.
The Western Kshatrap coins are reckoned to be the earliest coins bearing dates. The common copper
coins are the 'bull and hill' and the 'elephant and hill' types.
• From around 200 BC, many Greek adventures rode off with their armies to try their hand at conquering
their own little kingdoms in the fabled land of golden fortune, India. The early coins seem like they could
have come directly from Greece itself, with their diadems and helmets, Greek scripts and Greek goddesses,
and Greek names like Demetrios and Appollodotos. Over time, these Indo-Greek kings became ever more
Indian. They started worshipping the Buddha and Hindu gods like Vishnu and Krishna. In fact, the
earliest known depiction of Lord Krishna is on the coin of a King Agathokles, who depicts him in a
chiton (an ancient Greek dress worn by men), along with his chakra. They were the first to issue gold
coins in India. Hence, statement 2 is correct.
• Earliest Kushan coinage is generally attributed to Vima Kadphises. The Kushan coins generally depicted
iconographic forms drawn from Greek, Mesopotamian, Zorastrian and Indian mythology. Lord Shiva,
Buddha and Kartikeya were the major Indian deities portrayed. Kushan gold coins influenced
subsequent issues, notably those of the Guptas. Hence, statement 3 is correct.
• Gupta coinage (4th-6th centuries AD) followed the tradition of the Kushans, depicting the king on the
obverse and a deity on the reverse; the deities were Indian and the legends were in Brahmi. The earliest
Gupta coins are attributed to Samudragupta, Chandragupta II and Kumaragupta and their coins often
commemorate dynastic succession as well as significant socio-political events, like marriage alliances, the
horse sacrifice, etc (King and queen type of coin of Chandragupta-I, Asvamedha type, etc.), or for that
matter artistic and personal accomplishments of royal members (Lyrist, Archer, Lion-slayer etc.). Gupta
rulers issued many different types of gold coins. Hence, statement 4 is not correct.

Q 68.C
• The Harappan Script: The Harappans invented the art of writing like the people of ancient
Mesopotamia. Although the earliest specimen of the Harappan script was discovered in 1853 and the
complete script by 1923, it is yet to be deciphered.
o There are nearly 4000 specimens of Harappan writing on stone seals and other objects.
o Unlike the Egyptians and Mesopotamians, the Harappans did not write long inscriptions. Hence,
statement 1 is not correct.
o Most inscriptions were recorded on seals and contain only a few words. Hence, statement 2 is
correct.
o These seals may have been used by the propertied to mark and identify their private property. Altogether
we have about 250 to 400 pictographs, and in the form of a picture, each letter stands for some sound,
idea, or object.
o The Harappan script is not alphabetical but largely pictographic. Hence, statement 3 is correct.
o Attempts have been made to compare it with the contemporary scripts of Mesopotamia and Egypt, but
it is the indigenous product of the Indus region and does not indicate any connection with the scripts of
western Asia.

Q 69.C
• Tripura is one of the seven states in the northeastern part of India with a geographical area of 10,491 km2.
It is located in the south-west extreme corner of the north-eastern region, between latitudes 22°57' and
24°33' N and longitudes 91°10' and 92°20' E. The state is situated between the river valley of Myanmar and
25 www.visionias.in ©Vision IAS
Bangladesh, and is bounded by Bangladesh on the north, west, south and southeast; in the east, it has a
common boundary with Assam and Mizoram. The tropic of cancer passes through the state.
• Tripura is a landlocked state and its geographical
limits touch both national and international
boundaries. Its length of the international boundary line
with Bangladesh measures 839 km. Its national
boundaries with Assam and Mizoram measure 53 km and
109 km respectively.
• According to the census 2011, Out of total population,
73.83% of population lives in Urban area and 26.17%
lives in Rural area. There are 17.83% Scheduled Caste
(SC) and 31.76% Scheduled Tribe (ST) of total
population in Tripura. Tripura/Tripuri, Riang, Jamatia,
Noatia, Uchai, Chakma, Mog, Lushai, Kuki, Halam,
Munda, Kaur, Orang, Santal, Bhil, Bhutia, Chaimal,
Garo, Khasia, and Lepcha are the tribes situated in
Tripura.
• Humidity is generally high throughout the year. In the
summer season, the relative humidity is varied from 50
percent to 74 percent whereas, in the rainy season, it is over 85 percent.
• The Trishna Wildlife Sanctuary, 111 km from Agartala is famous for its rich variety of flora and
fauna. Occupying an area of 197.7 sq km, it was established in 1988. The vegetation of the sanctuary
consists of four types of forest – the tropical semi-evergreen forest, the east Himalayan lower Bhanar
Sal, the Moist mixed deciduous forest and the Savanah woodland.
• Some of the major attractions of the sanctuary are the Indian Gaur (Bison), Deer, Hoolock Gibbon, Golden
langur, Pheasants, Lalmukh Bandar, Wild Boar, Wild Cat and Leopard. It is also home to a great variety of
reptiles and different species of resident and migratory birds. Hence, option (c) is the correct answer.
• The sanctuary is also home to the last surviving populations of spectacle monkeys.

Q 70.A
• Anthrax is a zoonotic disease (could be transferred from animals to humans). Hence statement 1 is
correct.
• It is caused by the spore-producing bacterium Bacillus anthracis. Hence statement 2 is not correct.
• Reservoirs are grass-eating animals, and the spores can survive in the environment for decades.
• It is usually a disease of wild and domestic animals, including cattle, sheep, and goats.
• Human infection, while rare, does occur, which usually results from contact with infected animals or their
products.
• In 1922, the International Anthrax Commission has passed a resolution requiring the hairs and wools
used in brush-making, upholstering and textile industries be disinfected before they are handled
industrially.

Q 71.A
• Article 164 (1) of the Indian Constitution provides for the appointment of a minister in charge of tribal
affairs. It specifies the names of the states that are required to have a minister of tribal welfare. Tribal Affairs
Minister is required in the states of Jharkhand, Madhya Pradesh, Chhattisgarh, and Odisha as per the
94th Amendment Act of 2006. Hence statement 1 is correct.
• There is no mention in the Indian Constitution regarding the requirement of a minister of tribal welfare in
the fifth-schedule or sixth-schedule areas. Hence statement 2 is not correct.
• States having Scheduled areas (under the fifth schedule) must have Tribal Advisory Council.

Q 72.D
• After the global financial crisis of 2007-08, it was felt that banks ought to operate with a higher proportion
of their own, permanent capital as opposed to borrowed capital. This permanent capital is termed as Tier 1
capital. To shore up their Tier 1 capital, banks were allowed to raise a special class of bonds known as AT1
bonds from investors.
• AT1 bonds, short for Additional Tier 1 bonds, are unsecured bonds issued by banks. AT1 bonds, like other
bonds, pay regular interest. But they do not have a maturity date, as they are a permanent part of the bank’s
capital, akin to equity.

26 www.visionias.in ©Vision IAS


• In practice, however, banks do offer a call option on these bonds after five years, when they may or may
not redeem them. Apart from having no maturity date, AT1 bonds are different from vanilla bonds in other
respects too.
• The bank issuing them has the discretion to either reduce or completely skip their interest payout, if it is
making losses or at risk of falling short of capital needs. If the RBI believes that the bank is becoming short
of capital or unviable to operate, it can direct the principal on these bonds to be written off too. This makes
AT1 bonds far riskier than vanilla corporate bonds. Hence option (d) is the correct answer.
o Plain vanilla describes the simplest form of an asset or financial instrument. There are no frills,
no extras, and it can be applied to categories such as options or bonds.
• AT1 bonds are subordinate to all other debt and only senior to common equity. Mutual funds (MFs) were
among the largest investors in perpetual debt instruments.
• Recently, the Bombay High Court quashed the write-off of Additional Tier-1 (AT1) bonds worth Rs 8,400
crore issued by Yes Bank Ltd, bringing relief to investors.

Q 73.D
• The Chalukyas patronised both Saivism and Vaishnavism. They built temples for Siva and Vishnu.
o Brahmin groups were invited from the Gangetic regions and settled to perform regular pujas and
conduct festivals and ceremonies in the temples. Notable Chalukya rulers like Kirtivarman (566-
597), Mangalesa (597-609), and Pulakesin II (609-642) performed yajnas. They bore titles such as
parama-vaishana and parama-maheswara.
o Chalukyas gave prominent place to Kartikeyan, the war god.
o Chalukyas patronised heterodox sects also and lavishly donated lands to the Jain centres.
o According to Hiuen Tsang, there were many Buddhist centres in the Chalukya territory wherein
more than 5000 followers of the Hinayana and Mahayana sects lived.
• Chalukyan Architecture: Historically, in Deccan, Chalukyas introduced the technique of building temples
using soft sandstones as a medium. In Badami, two temples are dedicated to Vishnu and one each to Siva
and to the Jaina Tirthankaras. Their temples are grouped into two: excavated cave temples and structural
temples. Badami is known for both structural and excavated cave temples. Pattadakal and Aihole are
popular for structural temples.
• Aihole (Ayyavole): Built-in 634, Aihole, the headquarters of the famous medieval Ayyavole merchants’
guild was an important commercial center.
o About seventy temples are located in Aihole. The earliest stone-built temple is the Lad Khan
temple. Its unique trait is a stucco pillar with a big capital distinct from the northern style. A temple
dedicated to the goddess Durga was built on the model of Buddha Chaitya. It stands on a raised platform
in the form of a semi-circle.
o Another temple, dedicated to the same goddess is called Huccimalligudi, which is rectangular in
shape. Chalukyas also built Jain temples. Hence, statement 1 is correct.
o Megudi Jain temple is illustrative of the evolution of temple architecture under the Chalukyas. The
mandapa-type caves are preserved at Aihole.
• Badami (Vatapi)
o There are four caves in Badami. The largest cave temple built by Mangalesa is dedicated to
Vishnu. The reclining posture of Vishnu on the snake bed and Narasimha are exquisite examples
of Chalukya art. Hence, statement 2 is correct.
• Pattadakal
o Pattadakal was a center for performing royal rituals.
o The Virupaksha temple was built at the order of queen Lohamahadevi to commemorate the
conquest of Kanchipuram by her husband Vikramaditya II.
o The unique feature of the structural temple built by Rajasimha at Mamallapuram was adopted here by
the Chalukyas. Monuments are generally associated with the rulers who built them.
o However, here we also have signatures of the architects who conceived the edifices and the skilled
craftspeople who created them. The east porch of the Virupaksha temple has a Kannada inscription
eulogizing the architect who designed the temple. The architect was given the title Tribhuavacharya
(maker of the three worlds). Several reliefs on the temple walls bear signatures of the sculptors who
carved them. Hence, statement 3 is correct.
o At the south-eastern corner of the village is the Papanatha temple. Similar to the Virupaksha temple
in its basic plan, it has a shikhara in the northern style. The outer walls are richly decorated with many
panels depicting scenes and characters from the Ramayana. The eastern wall has a short Kannada
inscription, giving the name of the architect Revadi Ovajja, who designed the shrine.

27 www.visionias.in ©Vision IAS


Q 74.B
• State-run power giant NTPC REL, will set up the country's first green Hydrogen Mobility project in
Ladakh. Hence, option (b) is the correct answer.
• The objective of the pilot project is to deploy five fuel cell buses running around Leh. The government of
India conceptualised a Green Hydrogen Policy for compliance by the stakeholders in continuation to the
National Hydrogen Mission.
• Green hydrogen is produced by electrolysis of water using renewable energy (like Solar, Wind) and has a
lower carbon footprint. It is one of the best Zero Emission solutions. It is completely environment friendly
with no tailpipe emissions other than water.
• Hydrogen fuel cells make little noise means that they can be used in challenging contexts, such as in hospital
buildings.

Q 75.C
• Frozen foods have outstanding quality and nutritive value. Indeed, some frozen vegetables, such as green
peas and sweet corn, may be superior in flavour to fresh produce. The high quality of frozen foods is
mainly due to the development of a technology known as the individually quick-frozen (IQF) method.
• IQF is a method that does not allow large ice crystals to form in vegetable cells. Hence, statement 2 is
correct.
• Also, since each piece is individually frozen, particles do not cohere, and the final product is not frozen
into a solid block.
• Various freezing techniques are commonly used in the preservation of vegetables. These include blast
freezing, plate freezing, belt-tunnel freezing, fluidized-bed freezing, cryogenic freezing, and
dehydrofreezing. The choice of method depends on the quality of the end product desired, the kind of
vegetable to be frozen, capital limitations, and whether or not the products are to be stored as bulk or as
individual retail packages.
• This individual quick frozen process allows the ice crystals that form inside the cells of the food tissues
to be very small, which does not allow the cell walls of the food to break. Therefore, when defrosting
the product there is no leakage of cellular fluids, which is where the nutrients of the food are found,
and at the same time, it guarantees that the original texture and flavor are maintained. Hence,
statement 1 is correct.

Q 76.A
• Recently, the Ministry of Power has notified Electricity (Promoting Renewable Energy Through
Green Energy Open Access) Rules, 2022 in order to further accelerate our ambitious renewable energy
programmes, with the objective of ensuring access to affordable, reliable, sustainable and green energy for
all. Hence statement 2 is not correct.
• The salient features and benefits to common consumers from 'Green Energy Open Access' are as follow:
o Reduction in the limit of Open Access Transaction from 1 MW to 100 KW for green energy.
o Uniform renewable purchase obligation on all obligated entities in the area of distribution licensees.
o Green certificates for consumers if they consume green power. Hence statement 1 is correct.
o Cross subsidy surcharge and additional surcharge shall not be applicable if green energy is utilized for
the production of green hydrogen and green ammonia.
• The reduction of Open Access Transaction limit from 1 MW to 100 kW and appropriate provisions
for cross-subsidy surcharge, additional surcharge will incentivize the common consumers to get
Green Power at reasonable rates.

Q 77.D
• Recently, the Reserve Bank of India Governor met heads of public and private sector lenders to discuss
various issues including slower deposits growth compared to credit and asset quality. As per the RBI’s latest
weekly data for scheduled commercial banks, aggregate deposits have grown 8.2% in comparison to 11.4%
on a year-over-year basis whereas credit off-take has jumped 17% in comparison to a 7.1% increase on a
YoY basis.
• Reasons for slower deposit growth vis-a-vis credit growth:
o Depositors are parking their funds (savings) into the capital market instruments like share,
debenture/bonds and mutual funds in search of better returns. The migration of investors from banks
to the stock market affected the deposit growth of banks.
o High inflation resulting in small savings is affecting deposit growth. An increase in the households’
inflation expectations impacts households savings in debt-based instruments like bank term-
deposits negatively.
28 www.visionias.in ©Vision IAS
o RBI has been rising the repo rate to contain inflation in the economy. Banks passed on higher rates
through loan portfolios, most of which were at floating rates. While banks have been fast in passing
on the hikes in repo rates to borrowers, the increase in deposit rates has not kept pace. An increase
in the deposit rate may attract more deposits.
• Hence option (d) is the correct answer.

Q 78.C
• The Banni grassland in Gujarat’s Kachchh district is one of the largest grasslands in the Indian
subcontinent with an area of over 2500 sq.km. It is a region that is both socio-culturally unique and
ecologically valuable. Hence statement 1 is not correct.
• Recently the state sought to restore at least 76,000 hectares of this 2,500 sq km grassland which is a high-
biodiversity area. It was declared a Protected Forest in 1955, under the Indian Forest Act, 1927.
• Besides having 40 species of grass and 99 species of flowering plants, Banni is also home to the Indian
wolf, jackal, Indian fox, desert fox, desert cat, caracal, hyena, chinkara, Nilgai, wild boar, Indian hare,
common monitor lizard — and the cheetah before it became extinct.
o Banni also has 273 bird species and in years of good rainfall, is home to thousands of migratory birds.
o The stump-tailed macaque, also called the bear macaque, is an old World monkey native to
Cambodia, southwest China, northeast India, Laos, Myanmar, northwest Peninsular Malaysia,
Thailand, and Vietnam.
o Stump-tailed macaque habitat is limited to northeast India. Hence statement 2 is not correct.
• Over the years, the landscape of Banni has shown drastic changes with the deterioration of the
grassland taking place due to “heavy uncontrolled grazing”, widespread ingress of Prosopis Juliflora
(a harmful exotic tree species), dams constructed on rivers flowing towards Banni, periodic occurrence of
droughts and continuous increase in soil salinity. Hence statement 3 is correct.
• The mainstay of the restoration project is the removal of the alien species Prosopis Juliflora , which
incidentally was introduced to the area by the forest department in the 1960s to stop the ingress of the salt
flats.
• The grasslands have native trees like Acacia nilotica, Salvadora persica and Capparis decidua which are
protected under Section 26 in the Indian Forest Act, 1927. These have been destroyed to grow fodder. Banni
has sensitive soil ecology where the sweet soil rests on salinity only 2 to 3 metres below the ground and any
disturbance of the soil brings up salinity destroying the rich productivity of the land.

Q 79.B
• A sudden change on August 29, 2022 in the colour and smell of Lake Kuk, in north-west Cameroon, has
caused anxiety and panic among the local residents.
• Fears are driven by an incident that happened 36 years ago at Lake Nyos, just 10km away. On 21 August
1986, Lake Nyos emitted lethal gases (mainly carbon dioxide) that suffocated 1,746 people and around
8,300 livestock. It wasn’t the first incident like this.
• Both Kuk and Nyos are crater lakes located in a region of volcanic activity known as the Cameroon Volcanic
Line. And there are 43 other crater lakes in the region that could contain lethal amounts of gases.
• Hence, option (b) is the correct answer.

Q 80.A
• Animal Welfare Board of India (AWBI) has issued advisories regarding stray dogs and pet dogs
because of rising atrocities against dogs, feeders of dogs, and caregivers.
o The Central Government has framed the Animal Birth Control (Dog) Rule, 2001 which is to be
implemented by the local authority to control the population of stray dogs. The main focus of the rules
is on anti-rabies vaccination of stray dogs and neutering of stray dogs as means of population
stabilization. However, it has been noticed that there is a lack of proper implementation of Animal Birth
Control (Dog) Rules, 2001 by the Municipal Corporation / Local Bodies and instead attempts are made
for relocation of dogs from the urban areas.
• Animal Welfare Board of India (AWBI) is a Statutory Body established under the Prevention of Cruelty to
Animal Act, 1960 (PCA Act). Hence statement 1 is correct.
• The AWBI is an advisory body to the Central Government and State Government and also looks after the
matter of implementation of the PCA Act, 1960 and the Rules framed under this Act. Hence statement 2
is not correct.
• The Animal Welfare Board of India (AWBI), is headquartered in Ballabhgarh in Haryana state. The
AWBI headquarters were previously situated in Chennai. Hence statement 3 is not correct.

29 www.visionias.in ©Vision IAS


Q 81.C
• Centripetal and centrifugal are the forces experienced by rotating objects. The centripetal force keeps
an object moving in a circle and is always pointed toward the center of that circle.
• For instance, the gravitational force of the sun is a centripetal force that keeps the Earth orbiting around it.
Meanwhile, the centrifugal force is an apparent outward force on an object that is moving in a circle. An
example of centrifugal force would be the sensation you have when riding a merry-go-round that makes
you want to fly outwards.
• A centrifuge, used in diagnostic laboratories for blood tests, is a device that spins test tubes containing
the samples. As the device whirls, the centrifugal force causes heavier components, like red blood cells, to
settle at the bottom of the container, with the platelets above them, then the white blood cells and the plasma
at the very top, according to their relative densities.
• The art of pottery depends on centrifugal and frictional forces. A potter puts a lump of clay at the centre
of a flat wheel. The wheel is rotated at high speeds. The centrifugal force will push the clay outward, forming
a round shape. The frictional force between the hand and the clay shapes the pot along with the centrifugal
force.
• The washing machines exhibit a spinning mechanism and use centrifugal force to push the clothing,
water, and dirt particles toward the drum’s outer side. The pouch on the inner side of the container
subsequently gathers the dirt, and the garments are thoroughly cleaned. The fundamental principle here is
the centrifugal force acting on water.
• The motion of the earth and the other planets around the sun is strictly curvilinear in nature. However,
these celestial bodies do not leave their orbits and tend to keep revolving within their respective paths. A
significant amount of centripetal force that is directed toward the center gets developed and is responsible
to keep the planets and other celestial bodies in their respective orbits. Here, a balance is maintained
between the force of gravitation of the sun and the centripetal force. Hence, the resultant force is
called centripetal gravitational force.
• Hence option (c) is the correct answer.

Q 82.C
• Ranjit Singh died in June 1839, and, with his death, the process of the decline of the Sikh empire began.
• The outbreak of the first of Anglo-Sikh wars has been attributed to the action of the Sikh army
crossing the river Sutlej on December 11, 1845. This was seen as an aggressive maneuver that
provided the English with the justification to declare war. Hence statement 1 is correct.
• The other causes were, may be listed as follows:
o the anarchy in the Lahore kingdom following the death of Maharaja Ranjit Singh resulted in a power
struggle for domination between the court at Lahore and the ever-powerful and increasingly local army
o suspicions amongst the Sikh army arising from English military campaigns to achieve the annexation
of Gwalior and Sindh in 1841 and the campaign in Afghanistan in 1842
o the increase in the number of English troops being stationed near the border with the Lahore kingdom.
• The war began in December 1845, with 20,000 to 30,000 troops on the British side, while the Sikhs had
about 50,000 men under the overall command of Lal Singh. But the treachery of Lal Singh and Teja Singh
caused five successive defeats to the Sikhs and finally, Lahore fell to the British forces on February 20,
1846, without a fight.
• Treaty of Lahore (March 8, 1846) The end of the first Anglo-Sikh War forced the Sikhs to sign a
humiliating treaty on March 8, 1846. The main features of the Treaty of Lahore were as follows:
o War indemnity of more than 1 crore rupees was to be given to the English.
o The Jalandhar Doab (between the Beas and the Sutlej) was annexed to the Company’s dominions.
o A British resident was to be established at Lahore under Henry Lawrence.
o The strength of the Sikh army was reduced.
o Daleep Singh was recognized as the ruler under Rani Jindan as regent and Lal Singh as wazir.
o Since the Sikhs were not able to pay the entire war indemnity, Kashmir, including Jammu, was sold to
Gulab Singh and he was required to pay 75 lakh rupees to the Company as the price. The transfer of
Kashmir to Gulab Singh was formalized by a separate treaty on March 16, 1846.
• Governor General at the time was Henry Hardinge (1844-1848). Hence statement 3 is correct.
• Treaty of Gandamak (May 1879) was a treaty signed after the Second-Anglo-Afghan War. Hence
statement 2 is not correct.
o It provided that:
✓ the Amir conduct his foreign policy with the advice of the Government of India
✓ a permanent British resident be stationed at Kabul and
✓ the Government of India gave Amir all support against foreign aggression and an annual subsidy.
30 www.visionias.in ©Vision IAS
Q 83.D
• Fungi possess the biochemical and ecological capacity to degrade environmental organic chemicals and to
decrease the risk associated with metals, metalloids and radionuclides, either by chemical modification or
by influencing chemical bioavailability. This is called Mycoremediation.
• Fungi have been shown to play a significant role in the bioremediation of a variety of pollutants such as
POPs, textile dyes, petroleum hydrocarbons, pulp and paper industry effluents, leather tanning effluents.
o Filamentous fungi like Aspergillus, Curvularia, Acrimonium, Pithium, have been studied for their
metal tolerance ability.
o Pleurotus species have high biosorption potential due to their extensive biomass, i.e. mycelial
production. The genus has been reported to accumulate high levels of heavy metals. White-rot
fungi Pleurotus ostreatus has been reported to degrade model Polycyclic aromatic hydrocarbons
(PAHs) in solid-state fermentation (SSF) during growth on agro-industrial wastes, such as orange peels.
o Trichoderma viride is used to degrade nitrogenous explosives, such as TNT (Trinitrotoluene).
Trichoderma species also showed potential in bioremediation of hydrocarbons.
• Some other fungi that have shown bioremediation potential are Doratomyces nanus, Myceliophthora
thermophila, Mortierella, Exophiala xenobiotic, Rhizopus oryzae, Pleurotus eryngii, Phanerochaete
chrysosporium etc.
• Hence option (d) is the correct answer.

Q 84.C
• The Oort Cloud is the most distant region of our solar system. It lies beyond the Kuiper Belt.
• Unlike the orbits of the planets and the Kuiper Belt, which lie mostly in the same flat disk around the Sun,
the Oort Cloud is believed to be a giant spherical shell surrounding the rest of the solar system.
• It is like a big, thick-walled bubble made of icy pieces of space debris the sizes of mountains and sometimes
larger.
• The Oort Cloud might contain billions, or even trillions, of objects. It is divided into two regions: a disc-
shaped inner Oort cloud (or Hills cloud) and a spherical outer Oort cloud. Both regions lie beyond the
heliosphere.
• Scientists suspect that the Oort Cloud is the source of most of the long-period comets including comet
C/2013 A1 Siding Spring which made a very close pass by Mars in 2014 and will not return for about
740,000 years.
• The inner edge of the Oort Cloud is thought to be between 2,000 and 5,000 AU from the Sun. The outer
edge might be 10,000 or even 100,000 AU from the Sun — that's one-quarter to halfway between the Sun
and the nearest neighboring star.
• Hence option (c) is the correct answer.

Q 85.D
• Indian Councils Act of 1909 is also known as the Morley-Minto Reforms (Lord Morley was the then
Secretary of State for India and Lord Minto was the then Viceroy of India).
• The features of this Act were as follows:
o It considerably increased the size of the legislative councils, both Central and provincial. The number
of members in the Central legislative council was raised from 16 to 60. The number of members in the
provincial legislative councils was not uniform.
o It retained an official majority in the Central legislative council but allowed the provincial
legislative councils to have a nonofficial majority. Hence statement 1 is correct.
o It enlarged the deliberative functions of the legislative councils at both levels. For example, members
were allowed to ask supplementary questions, move resolutions on the budget, and so on.
o It provided (for the first time) for the association of Indians with the executive councils of the
Viceroy and Governors. Satyendra Prasad Sinha became the first Indian to join the Viceroy’s
executive council. He was appointed as the Law Member. Hence statement 2 is correct.
o It introduced a system of communal representation for Muslims by accepting the concept of a ‘separate
electorate’. Under this, the Muslim members were to be elected only by Muslim voters.
o It also provided for the separate representation of presidency corporations, chambers of
commerce, universities, and zamindars. Hence statement 3 is correct.
• But the reformed councils still enjoyed no real power and remained mere advisory bodies. They also did
not introduce an element of democracy or self-government. The undemocratic, foreign and exploitative
character of British rule remained unchanged.
• The Moderates and the country as a whole were disappointed by the ‘constitutional’ reforms of 1909.

31 www.visionias.in ©Vision IAS


Q 86.C
• Gombeyatta, Karnataka:
o The string puppets of Karnataka are called Gombeyatta, whereas, Tholu Bommalata is the
shadow puppet theatre tradition of the state of Andhra Pradesh. Hence statement 1 is not correct.
o They are styled and designed like the characters of Yakshagana, the traditional theatre form of the
region.
o The Gombeyatta puppet figures are highly stylized and have joints at the legs, shoulders, elbows, hips,
and knees.
o These puppets are manipulated by five to seven strings tied to a prop. Some of the more complicated
movements of the puppet are manipulated by two to three puppeteers at a time.
o Episodes enacted in Gombeyatta are usually based on Prasangas of the Yakshagana plays. The music
that accompanies is dramatic and beautifully blends folk and classical elements.
• Ravanachhaya, Odisha:
o The puppets are in one piece and have no joints. Hence statement 2 is correct.
o They are not colored, hence throwing opaque shadows on the screen.
o The manipulation requires great dexterity since there are no joints.
o The puppets are made of deer skin and are conceived in bold dramatic poses.
o Apart from human and animal characters, many props such as trees, mountains, chariots, etc. are also
used.
o Although Ravanachhaya puppets are smaller in size-the largest not more than two feet have no jointed
limbs, they create very sensitive and lyrical shadows.
• Yampuri, Bihar:
o The traditional Rod puppet of Bihar is known as Yampuri. These puppets are made of wood. Hence
statement 3 is not correct.
o Unlike the traditional Rod puppets of West Bengal and Orissa, these puppets are in one piece and have
no joints. As these puppets have no joints, the manipulation is different from other Rod puppets and
requires greater dexterity.
• The traditional rod puppet form of West Bengal is known as Putul Nautch.

Q 87.B
• The National Legal Services Authority (NALSA) has been constituted under the Legal Services Authorities
Act, 1987 to provide free Legal Services to the weaker sections of society and to organize Lok Adalats for
amicable settlement of disputes. The Chief Justice of India is the patron-in-chief of NALSA while the
second senior-most Judge of the Supreme Court of India is the Executive-Chairman. Currently, Justice D.
Y. Chandrachud, Chief Justice of India is the Patron-in-Chief. Hence statement 1 is correct.
• Collegium system: It is a system under which appointments and transfers of judges (of the higher judiciary)
are decided by a forum of the Chief Justice of India (CJI) and the four senior-most judges of the Supreme
Court (SC). It has no place in the Indian Constitution and has evolved through the judgments of the Supreme
Court. In case of a difference of opinion, the majority view will prevail. The CJI has no veto power in this
case. Hence statement 2 is not correct.
• A judge of the Supreme Court (including the Chief Justice of India) can be removed from his Office by an
order of the president. The President can issue the removal order only after an address by Parliament has
been presented to him in the same session for such removal. The address must be supported by a special
majority of each House of Parliament (ie, a majority of the total membership of that House and a majority
of not less than two-thirds of the members of that House present and voting). The grounds for removal are
two–proven misbehaviour or incapacity. The President of India removes (can remove) the Chief Justice of
India and not the Parliament. Hence statement 3 is not correct.

Q 88.A
• Adopted in 1987, the Montreal Protocol on Substances that Deplete the Ozone layer legally enforces
the phase-out of the production and use of ozone-depleting substances (ODS) - chemicals often used in
refrigeration, air-conditioning, foam manufacturing, aerosol production, and fire extinguishing. Hence
option (a) is not correct.
• The Protocol was designed to reduce the production and consumption of ozone-depleting substances in
order to reduce their abundance in the atmosphere and thereby protect the earth’s fragile ozone Layer. The
Protocol is to date one of the rare treaties to achieve universal ratification.
• The Montreal Protocol phases down the consumption and production of the different ODS in a step-wise
manner, with different timetables for developed and developing countries. Under this treaty, all parties have
specific responsibilities related to the phase out of the different groups of ODS, control of ODS trade, annual
32 www.visionias.in ©Vision IAS
reporting of data, national licensing systems to control ODS imports and exports, and other
matters. Developing and developed countries have equal but differentiated responsibilities, but most
importantly, both groups of countries have binding, time-targeted and measurable commitments.
• The Montreal Protocol uses three kinds of provisions as economic incentives to encourage participation and
compliance with the Protocol’s control regime; entry into force requirements, controls on trade with non-
parties, and research and technology transfer benefits. The Protocol promotes technology transfer to
developing countries, thereby offering economic incentives for developing countries to join and comply
through a network of 507 monitoring stations located all over the country.

Q 89.A
• The 1267 committee that was set up in 1999 (updated in 2011 and 2015) allows any UN member state to
propose adding the name of a terrorist or terror group to a consolidated list, maintained by the Committee,
that has affiliations to Al Qaeda and ISIS. Hence statement 1 is correct and statement 2 is not correct.
• Under the United Nations Security Council’s 1267 Al Qaeda Sanctions Committee regime, any U.N.
member state can propose the name of an individual or group to be designated as a terrorist. Decisions are
made by consensus in the 1267 Sanctions Committee, which comprises all members of the UNSC.
o A committee member can block blacklisting proposals by raising objections or applying a “technical
hold” on a proposal. An individual or entity listed as a terrorist is subjected to an assets freeze, travel
ban, and arms embargo.
• India has successfully proposed the listing of several terror entities in the past two decades, including
Pakistan-based Jaish-e-Mohammed (JeM) and Lashkar-e-Taiba.
o According to the rules, once a listing is proposed, it will be adopted into the list according to a
“no-objections” procedure, which means, if any member of the Committee, which comprises all
members of the UN Security Council, places a hold on the listing or objects outright to it, the
listing cannot be adopted.
• The Committee is bound to resolve all such pending issues within six months but can allow extensions,
meaning that technically at the end of the six-month period, the “holding” country has to decide whether to
accept the listing or place a permanent objection to it. However, in practice, many of the listing proposals
have had prolonged waits.
• Other important UNSC resolutions:
o The United Nations Security Council (UNSC) adopted Resolution 1373 on 28 September 2001 in
response to the terrorist attacks in the United States of America on 11 September 2001. Resolution 1373
requires Australia, as a UN Member State, to suppress terrorism by implementing targeted financial
sanctions in relation to persons involved in terrorist activities. Australia implements UNSC sanctions
by incorporating them into Australian law.
o The UN Security Council adopted resolution 1325 on women and peace and security on 31 October
2000. The resolution reaffirms the important role of women in the prevention and resolution of conflicts,
peace negotiations, peace-building, peacekeeping, humanitarian response and in post-conflict
reconstruction and stresses the importance of their equal participation and full involvement in all efforts
for the maintenance and promotion of peace and security.
✓ Resolution 1325 urges all actors to increase the participation of women and incorporate gender
perspectives in all United Nations peace and security efforts. It also calls on all parties to conflict
to take special measures to protect women and girls from gender-based violence, particularly rape
and other forms of sexual abuse, in situations of armed conflict. The resolution provides a number
of important operational mandates, with implications for Member States and the entities of the
United Nations system.
o The United Nations Security Council Resolution 2250 is a thematic resolution that deals with the
topic of youth from an international peace and security perspective. Recognizing the youth’s efforts in
peacebuilding, it provides a set of guidelines upon which policies and programs will be developed by
member states, the UN and civil society.
✓ This global policy framework, adopted by the UNSC in December 2015, explores how conflict
impacts young people’s lives and what must be done to mitigate its effects, as well as how youth
can be meaningfully included in creating peaceful communities.
✓ This resolution talks about five key action areas/pillars: Participation, Protection, Prevention,
Partnership and Disengagement and Reintegration. These five pillars and the resolution itself
promote a new narrative of young people with regard to peace and ensure legitimacy and
accountability.

33 www.visionias.in ©Vision IAS


Q 90.A
• Recent context: Tech startup OpenAI’s Dall-E, the AI-based image generator was made open to the
public and in November 2022, the company released ChatGPT, a new AI chatbot that appears to be
accurate in its responses.
• ChatGPT is the most recent AI tool released by OpenAI.The AI chatbot is capable of understanding
natural language and responding accordingly.
o It can provide effective answers to complex problems.
o Since the model is trained to predict what will come next, one can technically have a conversation with
it.
o As per OpenAI, ChatGPT is capable of answering follow-up questions, admitting its mistakes,
challenging incorrect premises, and rejecting inappropriate requests. However, it is still prone to
misinformation.
o ChatGPT is based on the newer GPT-3.5 model.
• OpenAI’s DALL-E is an image-generative model that has the ability to generate photo-realistic
images with nothing but a text prompt.
o It has its origin in GPT (Generative Pre-trained Transformer), which is an open-source artificial
intelligence program that can generate text, perform translations, answer questions, summarise
passages, and more.
o DALL-E uses the same model, except it uses it for generating images rather than text. It can combine
different objects, concepts, and styles while creating an image.
• LaMDA, or "Language Model for Dialogue Applications," is a language model built on a neural
network architecture developed by Google.
o The architecture produces a model that can be trained to read many words (a sentence or paragraph, for
example), pay attention to how those words relate to one another, and then predict what words it thinks
will come next and even what the following sequence of paragraphs might be.
o LaMDA is different from other language models because it is trained to understand the context
of the dialogue, not the text.
o As GPT-3 is focused on generating language text, LaMDA is focused on generating dialogue.
o This quality of understanding the context allows LaMDA to keep up with the flow of conversation
and provide the feeling that it’s listening and responding precisely to what is being said.
• Hence option (a) is the correct answer.

Q 91.B
• Article 267 (1) of the Indian constitution authorized the Parliament and state legislature to establish a
‘Contingency Fund of India’ and ‘Contingency Fund of the State’ into which amounts determined by law
are paid from time to time. Hence statement 1 is not correct.
• The contingency Fund of India is in the nature of an imprest (a fixed fund for a specific purpose) account
and is kept at the disposal of the President of India (by the Secretary to the Government of India, Ministry
of Finance, Department of Economic Affairs) to enable the government to meet unforeseen expenses
pending authorization by the Parliament. Hence, statement 2 is correct.
• The money is used to provide immediate relief to victims of natural calamities and also to implement any
new policy decision taken by the Government pending its approval by the Parliament. In all such cases after
the Parliament meets, a Bill is presented indicating the total expenditure to be incurred on the scheme/
project during the current financial year. After the Parliament votes on the Bill, the money already spent out
of the Contingency Fund is recouped/ withdrawn from the Consolidated Fund of India to ensure that
the corpus of the Contingency Fund remains intact.
• The corpus of the fund has been increased to Rs. 30,000 crores (as proposed in budget 2021-22, earlier it
was Rs. 500 crore) and can be enhanced from time to time by the Parliament.

Q 92.C
• Rivers are natural water bodies that flow from one place to another. However, at certain time
intervals, rivers change their course and move in different directions. There are some evident reasons
why this happens, and some unknown reasons are there that lead to the change of course. This change
of course is called as avulsion. Hence option (c) is the correct answer.
• Over a period of time, rivers tend to change their course due to the rocks present on the shores. It is quite
simple to understand that in plain areas, rivers will take the easiest and least resistant route. The pathway
that is easier to erode and cut through will be the path for the river. The speed of the flow of the river is
changed due to the obstruction in the river bed.

34 www.visionias.in ©Vision IAS


• This further gives rise to the meandering process, wherein the river flows faster, and there will be less
sediment deposition. When water is flowing faster, small curves will be formed. On the slower side of the
river, more sediments will be there and will cause the meander to grow.
• The bend will start growing with time, making the river’s slower side get slower and the faster side even
faster. The process will continue until the curves become sharp, allowing the river to cut through and re-
create a straight path. It is the time when the river will change its course and start flowing in a different
direction. Such change majorly happens at the time of flooding wherein the river has more energy to join
the ends. It will cut off from the main channel making the river flow independently.
• Alluvial fans are depositional landforms formed when streams flowing from higher levels break into
foot slope plains of low gradient. Normally, every coarse load is carried by streams flowing over mountain
slopes. This load becomes too heavy for the streams to be carried over gentler gradients and gets dumped
and spread as a broad low to high cone-shaped deposit called an alluvial fan.
• Rejuvenation occurs when the river’s base level falls (i.e. when the sea level falls). This can be a
consequence of either a fall in the amount of seawater or the land rising. Both are the result of the
comings and goings of ice ages. The effect on rivers is to produce features called “knick points” (which can
be seen as waterfalls and rapids), river terraces and incised meanders.
• River capture or stream piracy or stream capture is the natural diversion of the headwater of one
stream into the channel of another stream having greater erosional activity. A decapitated stream or
beheaded stream refers to the lower part of a stream whose headwaters have been captured by another
stream.

Q 93.A
• Initial public offering is the process by which a private company can go public by the sale of its stocks to
the general public. Companies can raise equity capital with the help of an IPO by issuing new shares to the
public or the existing shareholders can sell their shares to the public without raising any fresh capital.
• Follow-on Public Offer (FPO) is the issuance of securities for public trading by a company whose shares
are being traded in a stock exchange. Unlike in an IPO, the company that releases an FPO is already
listed on a stock exchange and later decides to offer equity to raise additional capital. Hence statement
1 is correct.
• FPO allows a company to raise additional funds through the issuance of new shares. This capital can be
used for a variety of purposes such as general corporate expenses, working capital, expansion, and debt
reduction. It also provides an opportunity for existing shareholders to increase their stake in the company.
• When compared, investing in FPO is less risky than investing in IPO. This is because data of the past
performance of the company in the share market is available when FPO is released. As a consequence of
‘reward for risk-taking behavior’, the profit-earning probability of investment is also less in an FPO than in
an IPO. Hence statement 2 is not correct.

Q 94.B
• The most severe part of the winter in Kashmir consists of three months. It is divided into three parts called
the Chilas:- The Chillai Kalan, the Chillai Khurd, and the Challai Bache.
• Chillai Kalan (Cold wave) is the harshest 40 days winter spell in the region of Kashmir triggered by
the western disturbance. It is a Persian term that means ‘major cold’. During Chillai-Kalan, the weather
in Kashmir valley continues to remain dry and cold with minimum temperatures hovering below the
freezing point. Hence option (b) is the correct answer.
o The 40-day period brings a lot of hardships for Kashmiris as the temperature drops considerably leading
to the freezing of water bodies, including the famous Dal Lake here.
o During these 40 days, the chances of snowfall are the highest and the maximum temperature drops
considerably. The minimum temperature in the Valley hovers below the freezing point.
• Chillai-Kalan is followed by a 20-day long Chillai Khurd (small cold) between January 30 and February
18 and a 10-days long Chillai Bachha (baby cold) from February 19 to February 28.

Q 95.D
• RIGHT TO FREEDOM
o Article 19 guarantees all citizens six rights. These are:
o Right to freedom of speech and expression.
o Right to assemble peaceably and without arms.
o Right to form associations or unions or co-operative societies.
o Right to move freely throughout the territory of India.
o Right to reside and settle in any part of the territory of India.
35 www.visionias.in ©Vision IAS
o Right to practice any profession or to carry on any occupation, trade, or business.
o Originally, Article 19 contained seven rights. But, the right to acquire, hold and dispose of the property
was deleted by the 44th Amendment Act of 1978. These six rights are protected against only state action
and not private individuals. Moreover, these rights are available only to the citizens and to shareholders
of a company but not to foreigners or legal persons like companies or corporations, etc.
o The State can impose ‘reasonable’ restrictions on the enjoyment of these six rights only on the grounds
mentioned in Article 19 itself and not on any other grounds.
o Freedom of Speech and Expression
✓ The State can impose reasonable restrictions on the exercise of the freedom of speech and
expression on the grounds of sovereignty and integrity of India, security of the state, friendly
relations with foreign states, public order, decency or morality, contempt of court, defamation, and
incitement to an offense.
o Freedom of Assembly
✓ The State can impose reasonable restrictions on the exercise of the right of assembly on two
grounds, namely, sovereignty and integrity of India and public order including the
maintenance of traffic in the area concerned. Under Section 144 of the Criminal Procedure Code
(1973), a magistrate can restrain an assembly, meeting, or procession if there is a risk of obstruction,
annoyance, or danger to human life, health, or safety or a disturbance of the public tranquillity or a
riot or any affray. Under Section 141 of the Indian Penal Code, an assembly of five or more persons
becomes unlawful if the object is (a) to resist the execution of any law or legal process; (b) to
forcibly occupy the property of some person; (c) to commit any mischief or criminal trespass; (d)
to force some person to do an illegal act; and (e) to threaten the government or its officials on
exercising lawful powers.
o Freedom of Association
✓ Reasonable restrictions can be imposed on the exercise of this right by the State on the
grounds of the sovereignty and integrity of India, public order, and morality. Subject to these
restrictions, the citizens have complete liberty to form associations or unions for pursuing lawful
objectives and purposes. However, the right to obtain recognition of the association is not a
fundamental right. The Supreme Court held that the trade unions have no guaranteed right to
effective bargaining or right to strike or right to declare a lockout. The right to strike can be
controlled by appropriate industrial law.
o Freedom of Movement
✓ The grounds for imposing reasonable restrictions on this freedom are two, namely, the
interests of the general public and the protection of the interests of any scheduled tribe. The
entry of outsiders in tribal areas is restricted to protect the distinctive culture, language, customs,
and manners of scheduled tribes and to safeguard their traditional vocation and properties against
exploitation. The Supreme Court held that the freedom of movement of prostitutes can be restricted
on the ground of public health and in the interest of public morals. The Bombay High Court
validated the restrictions on the movement of persons affected by AIDS. The freedom of movement
has two dimensions, viz, internal (right to move inside the country) and external (right to move out
of the country and right to come back to the country). Article 19 protects only the first dimension.
The second dimension is dealt with by Article 21 (right to life and personal liberty).
o Freedom of Residence
✓ The State can impose reasonable restrictions on the exercise of this right on two grounds,
namely, the interest of the general public and the protection of the interests of any scheduled
tribes. The right of outsiders to reside and settle in tribal areas is restricted to protect the distinctive
culture, language, customs, and manners of scheduled tribes and to safeguard their traditional
vocation and properties against exploitation. In many parts of the country, the tribals have been
permitted to regulate their property rights in accordance with their customary rules and laws. The
Supreme Court held that certain areas can be banned for certain kinds of persons like prostitutes
and habitual offenders. From the above, it is clear that the right to residence and the right to
movement are overlapping to some extent. Both are complementary to each other. Hence option
(d) is the correct answer.
o Freedom of Profession, etc.
✓ All citizens are given the right to practice any profession or to carry on any occupation, trade, or
business. This right is very wide as it covers all the means of earning one’s livelihood. The State
can impose reasonable restrictions on the exercise of this right in the interest of the general
public.

36 www.visionias.in ©Vision IAS


Q 96.D
• Ancient rock paintings identified in Bargur:
o Over 200 prehistoric paintings were recently identified at a cave in the forest area of Bargur hills, Erode
district, Tamil Nadu. Hence option (d) is the correct answer.
o A rock, 30 feet high and 250 feet in length, has paintings in red ochre and white. A hunting scene in
red displays human figures and a herd of deer standing opposite each other, while in some images,
deer of various sizes are presented.
o About 30 feet above the ground, an anthropomorphic painting, four feet high, was seen. It looks
similar to the Mother Goddess megalithic structures identified at T. Mottur village in Tiruvannamalai
district. Two deer and a few animals can be seen beneath the leg of the painting, which depicts the
deity blessing humans with resources for hunting.
o The elephant, tiger, and deer paintings are seen separately, while the middle row has the painting
of a human. It comprises a man standing on a crescent-shaped object and some other symbols, and
is considered to be a picture of a boat ride. The bottom layer depicts the racial conflict between
people, with three warriors opposing 10. The conflict between the racial groups is depicted in three
more places as ethnic groups fight to dominate or rule the region.
o The bottom row at the accessible height has hundreds of paintings, including those of humans,
animals, people dancing in groups, signs, and symbols of prosperity.

Q 97.B
• Reserve Bank of India has released its report ‘State Finances: A Study of Budgets’ on state government
budgets for 2022-23. It is an annual report by the Reserve Bank of India which provides data and analysis
of the fiscal position of State governments in India.
• Key findings of the report:
o The fiscal health of the States has improved from a sharp pandemic-induced deterioration in 2020-21
due to economic recovery and resulting high revenue collections. States’ gross fiscal deficit (GFD) is
budgeted to decline from 4.1 per cent of gross domestic product (GDP) in 2020-21 to 3.4 per cent
in 2022-23. Hence statement 2 is correct.
o For 2021-22, States budgeted a sharp rise in revenue receipts led by own tax and non-tax revenues and
grants from the Centre. States have budgeted for higher revenue receipts in 2022-23, driven primarily
by SGST, excise duties, and sales tax collections.
o The state debt-to-GDP ratio remains uncomfortably high. As per the report, the debt-to-GDP
ratio has fallen from 31.1 per cent in 2020-21 – a year when states had struggled to manage the
economic fallout of the pandemic — to 29.5 per cent in 2022-23. The Fiscal Responsibility and
Budget Management review committee, headed by N K Singh, had recommended a debt-to-GDP ratio
of 20 per cent for states. A high debt-deficit burden leaves little room for states to manoeuvre when
faced with the next economic shock. Hence statement 1 is not correct.
o State governments have also seen a significant expansion in their contingent liabilities. Contingent
liabilities here refer to the obligations of a state government to repay the principal and interest payments
in case a state-owned entity defaults on a loan. As per the report, the guarantees issued by state
governments have risen from Rs 3.12 lakh crore or 2 per cent of GDP in 2017 to Rs 7.4 lakh crore or
3.7 per cent of GDP.

Q 98.C
• The triple planetary crisis refers to the three main interlinked issues that humanity currently faces:
climate change, pollution and biodiversity loss. Each of these issues has its own causes and effects and
each issue needs to be resolved if we are to have a viable future on this planet.
o Climate change is the most pressing issue facing humanity today. Simply put, climate change refers
to long-term shifts in temperatures and weather patterns that in the long run will completely alter
the ecosystems that support life on the planet. Human activities are the main drivers of climate
change. Almost everything we do releases emissions, but energy use, industry, transport, buildings
and agriculture are the main causes for release of greenhouse gases to the atmosphere. The
consequences of climate change already today manifest through increased intensity and severity
of droughts, water scarcity, wildfires, rising sea levels, flooding, melting polar ice, catastrophic storms
and declining biodiversity.
o Pollution is the largest cause of disease and premature death in the world, with more than seven million
people dying prematurely each year due to pollution. Incredibly, nine out of ten people worldwide
breathe air that contains levels of pollutants that exceed WHO guidelines. Pollution is caused by
everything from traffic and factories to wildfires, volcanoes and mould. Another cause of pollution
37 www.visionias.in ©Vision IAS
is indoor household air pollution from cooking with polluting fuels and technologies which caused
an estimated 3.8 million deaths in 2016 alone.
o Biodiversity loss refers to the decline or disappearance of biological diversity, which includes
animals, plants and ecosystems. The reasons for biodiversity loss include everything from overfishing
to habitat loss (e.g. deforestation to make way for development) to desertification due to climate
change. Biodiversity is the baseline for everything on the planet – as in the end we are all
interlinked. Biodiversity loss impacts food supplies and access to clean water – without it we have no
future on our planet.
o Hence option (c) is the correct answer.

Q 99.B
• Vikram Sarabhai, in full Vikram Ambalal Sarabhai, (born August 12, 1919, Ahmadabad, India—
died December 30, 1971, Kovalam), Indian physicist and industrialist who initiated space research
and helped develop nuclear power in India.
• Sarabhai was born into a family of industrialists. He attended Gujarat College, Ahmadabad, but later shifted
to the University of Cambridge, England, where he took his tripos in natural sciences in 1940. World War
II forced him to return to India, where he undertook research in cosmic rays under physicist Sir
Chandrasekhara Venkata Raman at the Indian Institute of Science, Bangalore (Bengaluru). In 1945 he
returned to Cambridge to pursue a doctorate and wrote a thesis, “Cosmic Ray Investigations in Tropical
Latitudes,” in 1947. He founded the Physical Research Laboratory in Ahmadabad on his return to India.
• Establishing the Indian National Committee for Space Research in 1962, which was later renamed the Indian
Space Research Organization (ISRO), Sarabhai also set up the Thumba Equatorial Rocket Launching
Station in southern India. After the death of physicist Homi Bhabha in 1966, Sarabhai was appointed
chairman of the Atomic Energy Commission of India. Carrying forward Bhabha’s work in the field of
nuclear research, Sarabhai was largely responsible for establishing and developing India’s nuclear power
plants. He laid the foundations for the indigenous development of nuclear technology for defence
purposes.
• Dedicated to the use of all aspects of science and technology in general and to space applications in
particular as “levers of development,” Sarabhai initiated programs to take education to remote villages
through satellite communication and called for the development of satellite-based remote sensing of natural
resources.
• Sarabhai was awarded two of India’s highest honours, the Padma Bhushan (1966) and the Padma
Vibhushan (awarded posthumously in 1972). Hence, option (b) is the correct answer.

Q 100.C
• The decline of the imperial Mughals forced talented people to seek the patronage of newly established state
courts like Hyderabad, Lucknow, Jaipur, Murshidabad, Patna, Kashmir, etc.
• A distinct feature of the literary life of the 18th century was the growth of Urdu language and poetry. It
was the period of Urdu poets like Mir, Sauda, Nazir and Mirza Ghalib (19th century). Hence
• In south India, Malayalam literature flourished under the patronage of the Travancore rulers. Kanchan
Nambiar was a noted Malayalam poet. Hence pair 4 is correctly matched.
• The Tamil language was enriched by sittar poetry. Tayumanavar (1706-44), one of the best exponents
of sittar poetry, protested against the abuses of temple-rule and the caste system. Hence pair 3 is not
correctly matched.
• Heer Ranjha, the romantic epic in Punjabi literature, was composed by Warris Shah. Hence pair 1 is
not correctly matched.
• In Sindhi literature, Shah Abdul Latif composed Risalo, a collection of poems. Hence pair 2 is not
correctly matched.

Copyright © by Vision IAS


All rights are reserved. No part of this document may be reproduced, stored in a retrieval system or transmitted
in any form or by any means, electronic, mechanical, photocopying, recording or otherwise, without prior
permission of Vision IAS.
38 www.visionias.in ©Vision IAS
INDIAN COPIERS SYSTEMS

ALL KIND STUDY MATERIAL OF


IAS, PCS, SSC, IGNOU, JUDICIARY
ETC.
ALL TEST SERIES AVAILABLE

CONTACT US AT:
Telegram or Contact - 9953078429
or
Call at 9818130472

011- 44786293
EMAIL US:
Indiancopiers2018@gmail.com
Also Visit at : www.indiancopiers.com
COURIER FACILITY ALSO AVAILABLE ALL OVER INDIA

You might also like